» » 3 этап сестринского процесса включает в себя. Основные этапы сестринского процесса: планирование сестринской помощи

3 этап сестринского процесса включает в себя. Основные этапы сестринского процесса: планирование сестринской помощи

Тема занятия : «Планирование ухода»

1. Продолжительность занятия : 4 академических часа

2. Цель занятия :

Овладеть на доклиническом уровне профессиональными компетенциями :

§ ПК 5.1 Эффективно общаться с пациентом и его окружением в процессе профессиональной деятельности

§ ПК 5.2 Соблюдать принципы профессиональной этики

§ ПК 5.5 Оформлять медицинскую документацию

§ ПК 5.6 Оказывать медицинские услуги в пределах своих полномочий.

§ ПК 5.7 Обеспечивать инфекционную безопасность.

§ ПК 5.8 Обеспечивать безопасную больничную среду для пациентов и персонала.

§ ПК 5.12 Осуществлять сестринский процесс

Овладеть на доклиническом уровне общими компетенциями :

§ ОК 1. Понимать сущность и социальную значимость своей будущей профессии, проявлять к ней устойчивый интерес.

§ ОК 2. Организовывать собственную деятельность, исходя из цели и способов ее достижения, определенных руководителем

§ ОК 3. Анализировать рабочую ситуацию, осуществлять текущий и итоговый контроль, оценку и коррекцию собственной деятельности, нести ответственность за результаты своей работы.

§ ОК 4. Осуществлять поиск информации, необходимой для эффективного выполнения профессиональных задач.

§ ОК 5. Использовать информационно-коммуникационные технологии в профессиональной деятельности.

§ ОК 6. Работать в команде, эффективно общаться с коллегами, руководством, потребителями.

3. Порядок проведения и примерное распределение времени :

Организационная часть - 5 минут

Мотивация новой темы - 5 минут

Контроль усвоения материала по теме предыдущего занятия - 25 минут

Контроль исходного уровня базисных знаний и умений, необходимых для изучения темы - 10 минут



Изучение данного материала - 25 минут

Самостоятельная работа студентов - 70 минут

Контроль эффективности обучения - 20 минут

Подведение итогового занятия - 15 минут

Домашнее задание - 5 минут

4. Контроль усвоения знаний и умений предыдущего занятия : «Постановка целей»

5. Оснащение занятия по данной теме:

Таблица: выполнение плана по уходу.

Раздаточный материал из «Стандартов»

Информационный материал из журналов: « Медицинская сестра», «Сестринское дело».

7. Контрольные вопросы по данной теме:

  1. Понятие о плане ухода
  2. Значение плана по уходу за пациентом
  3. Индивидуальный план сестринского ухода
  4. Типы сестринских вмешательств
  5. Дайте определение «Стандарт сестринской практики
  6. Перечислите независимые сестринские вмешательства
  7. Перечислите взаимозависимые сестринские вмешательства

8. Задание для закрепления темы:

работа студентов с дидактическим материалом.

Тестовый контроль:

1 вариант

1.Установите соответствие.

виды диагностики: перечень проблем:

1) потенциальный физиологический А. перелом позвоночника

диагноз Б. недоверие медперсоналу

2) сестринский диагноз В. боль в спине

3) медицинский диагноз Г. страх смерти

Д. социальная изоляция

Е. риск ухудшения состояния из-за

Игнорирования своей болезни

Ж. риск усугубления отеков из-за

погрешностей в диете

2. В конце II этапа медсестра:

а) формулирует цели ухода

б) устанавливает приоритетные сестринские проблемы

3. Понятие сестринского диагноза или проблемы пациента впервые появилось в:

а) Японии

в) Голландии

г) Англии

4. Установите последовательность этапов сестринского процесса:

а) оценка

б) сестринская диагностика

в) выполнение

г) сбор данных

д) планирование

5. Установите соответствие:

Этапы сестринского процесса: действия медсестры на этом этапе:

1. планирование А. формулирование целей ухода

Б. интерпретация потребностей,

2. диагностика удовлетворение которых нарушено

В. Формулирование сестринских проблем

3.оценка Г. оказание непосредственной помощи пациенту

Д. коррекция ухода

6. Во время пребывания пациента на стационарном лечении сестринские диагнозы могут многократно меняться:

а) да, верно

б) нет, неправильно

7. Цель сестринского процесса:

а) диагностика и лечение заболевания

б) обеспечение приемлемого качества жизни в болезни

в) решение вопроса об очередности мероприятий ухода

г) активное сотрудничество с пациентом

8. Сестринский диагноз – это:

а) отражение сущности патологического процесса, происходящего в организме

б) ответная реакция пациента на болезнь и свое состояние

в) фиксация нарушений той или иной потребности

г) клиническое суждение медсестры, характеризующее состояние здоровья

II –Вариант

1 . Пример зависимого типа сестринского вмешательства:

б) промывание желудка

в) реанимационные мероприятия

г) профилактика пролежней

2. Пример независимого типа сестринского вмешательства:

а) постановка сифонной клизмы

б) организация досуга

в) назначение лечебного стола и режима двигательной активности

г) раздача лекарств

3. Установите соответствие:

1.сестринский психологический А. грипп

диагноз Б. социальная изоляция

2.медицинский диагноз В. дефицит знаний о рациональной

лечебной схеме

3.сестринский потенциальный диагноз Г. недержание кала

4. В центре любой сестринской проблемы всегда лежит нарушение удовлетворения одной или

нескольких человеческих потребностей:

а) да, верно

б) нет, неправильно

5. Установите соответствие:

виды диагностики: перечень диагнозов:

1. медицинский диагноз А. страх перед операцией

2. сестринский физиологический Б. запоры

диагноз В. Ангина

3.потенциальная психологическая Г. риск суицидальной попытки

проблема

6 . На III – этапе сестринского процесса цели и план ухода формулируется отдельно для

каждой приоритетной проблемы:

а) да, верно

б) нет, неправильно

7 . Установите соответствие.

термин: содержание термина:

1. план сестринского А.письменное руководство по

вмешательства индивидуальному уходу за пациентом

Б. документ, отражающий все пять

этапов сестринского процесса

2. сестринская история

болезни В. перечень всех действий медсестры,

обеспечивающих качественный уход

по определенной сестринской проблеме.

8. В конце III – этапа медсестра:

а) формулирует цели ухода

б) устанавливает приоритетные проблемы

в) знакомится с медицинской историей болезни

г) согласовывает цели и план ухода с пациентом

9. Студент должен знать:

1. понятие о плане ухода

2. значение плана по уходу за пациентом

3. понятие стандартного и индивидуального плана сестринского ухода

10.Студент должен уметь:

Определить объем сестринских вмешательств

Использовать стандартные планы ухода при написании индивидуального плана

сестринских вмешательств;

Согласовывать цели и план ухода за пациентом и его семьей;

Документировать планирование ухода в сестринской документации.

11. Задание для самоподготовки к следующему занятию

Сестринская диагностика - II этап сестринского процесса

Цели сестринского диагноза - проанализировать результаты обследования и определить, с какой проблемой здоровья сталкивается пациент и его семья, а также наметить план сестринского ухода.


После завершения обследования пациента медсестра переходит к установлению сестринского диагноза. Греческое слово "распознавание, определение" для врача означает установление причин страдания на основе выявления симптомов.

Сестринский диагноз – это продуманный вывод, основанный на анализе и интерпретации информации, полученной при обследовании, он дискутируется на реакциях пациента, связанных со здоровьем, а не на распознавании болезней.

Чтобы понять смысл и важность сестринского диагноза, необходимо знать эволюцию сестринского диагноза.

Обсуждение этой проблемы началось в 30-е годы в США. В медицинской литературе, посвященной работе медсестер, содержится много определений "диагностики медсестры". Было опубликовано множество статей "за" и "против" применения сестринского диагноза. Эти определения изменялись по мере того, как термин "диагностика медсестры" получал все большее понимание у профессионалов. Однако некоторые общие компоненты этих определений включают понятие "ухода за больными", "пациент и проблемы состояния здоровья". Помимо этого в каждом определении подразумеваются клиническая оценка и принятие решений.

В 80-е годы активность в пользу сестринской диагностики возросла, ив 1991г. сестринский диагноз был включен в Стандарты клинической сестринской практики (США).В чем же различие между медицинским диагнозом и диагнозом медсестры: (таблица № 4)

Медицинский (врачебный) диагноз - это определение состояния заболевания, основанное на специальной оценке физических признаков, симптомов, истории заболевания. Медицинский диагноз фокусируется на распознавании болезней.

Сестринский диагноз - это формулировка о действительной или потенциально возможной реакции пациента на заболевание (проблему с состоянием здоровья), которое медицинская сестра компетентна лечить. Сестринский диагноз отражает уровень состояния здоровья пациента или реакцию на болезнь или патологический процесс. Диагноз врача и диагноз медсестры устанавливаются на основании данных физиологического, психологического, социально-культурного, духовного и др. показателей обследования пациента.

Цели и задачи врачебной диагностики - определить заболевание и назначить лечение.

Цели сестринского диагноза - проанализировать результаты обследования и определить, с какой проблемой здоровья сталкивается пациент и его семья, а также наметить план сестринского ухода.

Задача диагностики медсестры - разработка индивидуального плана по уходу за больным с тем, чтобы пациент и его семья смогли бы адаптироваться к изменениям, возможным из-за возникших проблем со здоровьем.

Установление сестринского диагноза - это установление проблем пациента, связанных со здоровьем.

Сестринские диагнозы могут быть установлены пациенту, семье, общине и т.д. и учитывают физические, интеллектуальные, эмоциональные (психологические), социальные и духовные факторы, выявленные в ходе обследования.

Структура сестринского диагноза

Описание ответной реакции пациента на болезнь

описание возможной причины такой реакции

Таблица № 3

Второй этап сестринского процесса - сестринский диагноз - предусматривает проведение следующих мероприятий:

I . Обработка информации, полученной при обследовании

Медицинская сестра должна быть уверена, что данные обследования соответствуют определенной диагностической мерке (стандарту, эталону).

Например, спрашивая пациента о характере боли, мы получаем субъективную информацию. Однако пальпация больного места и искаженное от боли лицо пациента - это объективная информация.

Невнимательность, торопливость, безответственность медсестры может привести к нежелательным ошибкам. Эти ошибки могут возникнуть на любом этапе сестринского процесса: при обследовании и установлении сестринского диагноза, составлении плана сестринского ухода, при практической реализации плана и оценке результатов. Американские ученые Поттер П. и Перри А. рекомендуют некоторые способы, позволяющие избежать диагностических ошибок:

    Определить реакцию пациента на болезнь.

    Определить диагностическую формулировку.

    Установить причину, излечимую в процессе ухода за больным.

    Определить потребность пациента в том или ином курсе лечения или анализе.

    Выяснить реакцию пациента на аппаратуру.

    Уяснить проблему пациента, а не медсестры.

    Уяснить проблему пациента, а не вмешательства.

    Уяснить проблему пациента, а не цели.

    Избегайте наносящих вред формулировок.

    Обозначайте в диагностической формулировке только одну проблему пациента.

П. Выявление проблем пациента

После обработки информации медсестра устанавливает проблемы пациента, связанные со здоровьем.

Проблемы могут быть:

    физические ( физиологические )

    психологические

    социальные

    духовные

Например: В кардиологическое отделение поступила больная 70 лет с выраженной экспираторной одышкой и головной болью, которая появляется у нее на запах газа. Во время обследования больная была беспокойна, ее волновало ухудшение состояния здоровья, что одышка стала возникать каждый раз, когда женщина зажигала газовую плитку, и не проходила длительное время. Медсестре она сообщила также, что живет одна, и дома некому поливать цветы, переживала, что они засохнут, пока она в больнице. Женщина также была озабочена тем, что сейчас пост, и сможет ли она его соблюдать, находясь на лечении.

Таблица № 4

Определяем проблемы пациента.

    Физиологические - выраженная одышка, головная боль.

    Психологические - беспокойство по поводу ухудшения состояния здоровья (приступы участились), волнения о цветах (засохнут).

    Духовные - соблюдения поста.

III. Формулировка сестринских диагнозов

После установления проблем пациента необходимо сформулировать сестринские диагнозы. Ассоциация Американских медсестер (ААМ) выделила основные проблемы пациента:

    Ограничение самообслуживания.

    Нарушение сна, отдыха, питания, сексуальности, кровообращения и т.д.

    сердечная недостаточность

    нарушение питания (пониженное, повышенное и т.д.)

    пониженный газообмен

3. Боль (дискомфорт)

    хроническая боль

    хронический запор

    понос

4. Эмоциональная неустойчивость, связанная с болезнью, угрожающей здоровью и повседневной ЖИЗНИ.

    чувство страха

    чувство отчаяния, безнадежности

    беспокойство о ком или о чем-либо

    волнение по поводу...

    нерешительность в принятии решений

    дефицит желания следить за собой

5. Нарушение мыслительной деятельности

    нарушение речи

    неадекватная оценка своего состояния

    ситуационная потеря чувства собственного достоинства.

    Проблемы, связанные с жизненными циклами (рождение, смерть, стадии развития)

    Проблемы в сфере отношений

    семейные конфликты

    стрессовые ситуации

Это далеко не полный перечень формулировок сестринских диагнозов. Медсестра должна всегда помнить, что ее задача не определение болезни, а определение уровня состояния здоровья или реакции на болезнь или патологический процесс.

Какие сестринские диагнозы можно поставить пациентке в нашем примере?

    Выраженная одышка - нарушение функции дыхания, пониженный газообмен. Диагноз:

    "Беспокойство по поводу участившихся приступов одышки". Диагноз:

    "Волнение по поводу оставленных дома цветов". Диагноз:

    "Волнение по поводу соблюдения поста". Диагноз:

IV. Документирование

Все установленные сестринские диагнозы записываются в историю болезни - в карту сестринского процесса. Медсестра должна четко знать понятие, которым определила диагноз, чтобы не было разночтения, т.к. процесс ухода за больными осуществляется разными медсестрами.

Значение диагностики медсестры и ее применение в составлении плана ухода за больными:

Применение диагностики медсестры - это механизм, по которому устанавливается сфера деятельности медсестры по уходу за больным.

Сформулированные медсестрой диагнозы обеспечивают направление процесса планирования и выбор лечебного воздействия для достижения желаемых результатов. Предполагаемые результаты прогнозируются для каждого диагноза медицинской сестры. Диагноз медсестры и составляемый вслед за этим лечебный план ухода за пациентом помогают сообщать другим профессионалам проблемы пациента при помощи лечебного плана по уходу, консультаций, плана выписки и конференций по проблемам ухода за пациентами.

Диагностика медсестры облегчает передачу информации между медсестрами.

Первоначальный список диагнозов медсестры - это легкодоступный справочник для определения текущих потребностей пациента в лечении и медицинском уходе.

Сестринские диагнозы также побуждают медсестру развивать свои организационные навыки, ибо они помогают придавать большее значение нуждам пациента.

Сестринские диагнозы используются для составления записей медсестры о прогрессе состояния пациента, для написания направлений к врачу-специалисту, осуществлению эффективного лечения и ухода за пациентом при переводе его из одного отделения в другое, из одной больницы в другую. При планировании выписки пациентов с диагнозом медсестры представляют собой способ передачи информации и установлением, в каком лечении и уходе пациент все еще нуждается.

Сестринские диагнозы могут служить центром для гарантии качества, улучшения работы медсестры и совместных разборов.

Гарантия качества - это контроль и оценка качества и соответствия лечения и ухода за больным в сравнении с принятыми стандартами. Улучшение качества работы медсестры - это оценка со стороны профессионалов, того, как медсестра осуществляет свою практическую работу, повышает свою квалификацию, или участвует в научных исследованиях. Концентрируя внимание на диагнозе медсестры, проверяющий может определить, правильно ли было лечение и уход за больным, и проводилось ли оно в соответствии с принятыми на практике стандартами.

Сестра несет ответственность за свои суждения и действия на всех этапах сестринского процесса - от сбора данных и оценки состояний здоровья пациента до оценки эффективности и достижения поставленных целей.

III этап сестринского процесса - планирование сестринских вмешательств

Цель: сестринского планирования: исходя из потребностей пациента, выделить приоритетные задачи, разработать стратегию достижения поставленных целей, определить критерий их выполнения.


Сестринское обследование и формулировка сестринских диагнозов предполагают планирующую ступень в сестринском процессе. Планирование - категория, определяющая сестринское поведение при определении целей, сосредоточенных на пациенте и установлении стратегий для достижения целей. Во время планирования:

    устанавливаются приоритеты;

    определяются цели, ожидаемые результаты;

    выбираются меры по уходу за больными;

    устанавливаются возможные последствия;

    пишется план сестринского ухода.

1. Установление приоритетов

После установления специфических медсестринских диагнозов сестра определяет приоритеты в соответствии с серьезностью диагноза. Приоритеты по уходу устанавливаются для определения порядка, по которому осуществляется сестринское вмешательство, когда у пациента несколько проблем.

Установление приоритетов не является простым перечислением сестринских диагнозов в зависимости от их серьезности и психосоциальной значимости. Скорее, это метод, по которому пациент и сестра вместе устанавливают диагнозы, основываясь на желаниях, нуждах и безопасности пациента.


Таблица № 5

Основные психосоциальные потребности стоят на ступень выше по сравнению с нуждами безопасности. Потребности в любви, уважении и самовыражении может быть оказано меньше внимания. Сестра должна помнить о ситуациях, когда нет экстренных физических потребностей, но приоритет может быть отдан психологическим, социокультурным, развивающим и духовным потребностям пациента.

Поскольку у пациента несколько диагнозов, сестра не может приступить к их лечению одновременно, после их установления. Она выбирает в зависимости от срочности, природы предписанного лечения, взаимодействия между диагнозами. Приоритеты классифицируются как:

    первичные

    промежуточные вторичные

Таблица № 6

Первичный приоритет (ведущее значение) отдается сестринскому диагнозу (или состоянию пациента, его реакции), осуществление которого требует срочных мероприятий, так как от решения этой проблемы зависит состояние больного и дальнейшее лечение.

Обратимся к карте сестринского процесса.

Сестринскому диагнозу чувство «распирания» в области живота из-за длительной задержки стула отдается первичный приоритет, так как после обсуждения с пациентом медсестра сделала

вывод, что решение именно этой проблемы является и первоочередной задачей.

Промежуточный приоритет отдается таким диагнозам, которые не требуют экстренных мер.

В случае с нашим пациентом, это диагнозы:

    Высокий риск возникновения повторных запоров, связанных с нерациональным питанием и малоподвижным образом жизни.

    Дефицит заботы о своем здоровье.

    Высокий риск хронических заболеваний ЖКТ в результате длительных стойких запоров.

    Недостаток знаний о рациональном питании.

Вторичный приоритет - это потребности пациента, которые не имеют прямого отношения к заболеванию и прогнозу.

В нашем примере таких диапазонов нет, и здесь мнения пациента и медсестры совпадают. Но ситуация может быть и другой. Например, диагнозу "дефицит заботы о своем здоровье" можетбыть отдан вторичный приоритет, но это должно быть совместное решение пациента и медицинской сестры.

Запомните!

    1. Приоритеты по уходу устанавливаются для определения порядка, по которому

осуществляется сестринское вмешательство.

2.Это не просто перечисление сестринских диагнозов в зависимости от их серьезности и
психосоциальной значимости. Это метод, по которому пациент и медсестра вместе устанавливают диагнозы, основываясь на желаниях, нуждах и безопасности пациента.

2. Определение целей и ожидаемых результатов

Цели и ожидаемые результаты выделяются в процессе изучения поведения или реакции пациента, исходя из опыта сестры в уходе за больными. После обследования, установления диагноза и определения первичных нужд пациента сестра формулирует цели и ожидаемые результаты для каждого установленного диагноза вместе с пациентом.

Существуют две причины для написания целей и ожидаемых результатов.

Во-первых, цели и ожидаемые результаты дают направление для индивидуальной сестринской помощи.

Во-вторых, цели и результаты используются для определения степени эффективности помощи.

Цель проведения такой работы заключается в определении реакции пациента на сестринский уход.

На каждую цель и каждый ожидаемый результат должно быть отведено время для оценки. Отведенное время зависит от природы проблемы, этиологии, общего состояния пациента и установленного лечения.

Поскольку каждый пациент по-своему реагирует на различные жизненные ситуации, следовательно, сестринские диагнозы и цели по уходу будут уникальны (неповторимы, индивидуальны).

Цели, сосредоточенные на пациенте, предполагают активное участие пациента в их определении, в определении ожидаемых результатов и плане ухода.

Цели должны быть реальными и достижимыми.

Цели должны быть конкретными, не расплывчатыми, следует избегать общих формулировок («пациент будет чувствовать себя лучше», «пациент не будет чувствовать дискомфорта», «пациент будет адоптирован»).

Цель должна быть сформулирована в пределах сестринской, а не врачебной компетенции.

Цели должны иметь конкретные сроки их достижения.

Цель должна быть понятна пациенту, его семье, а также другим медицинским работникам.

Пациент должен быть максимально вовлечен в процесс планирования и реализации программ по защите его здоровья. Он имеет полное моральное право получать информацию, необходимую для принятия серьезных решений, содействовать оценке преимуществ и недостатков при выборе способов лечения, принимать, отказываться или продолжать лечение без принуждения. Каждая медсестра должна быть компетентна в вопросах моральных и юридических прав пациента и должна защищать и поддерживать эти права. Если пациент не способен принять самостоятельное решение, то необходимо найти того, кто смог бы это сделать (родственники, опекуны). Медсестра должна знать и те ситуации, в. которых права личности на независимость в сфере охраны здоровья могут временно отойти на второй план, чтобы защитить общество (например, если болезнь требует изоляции больного от общества или заболевший человек представляет прямую угрозу окружающим - острые формы психозов, особо опасные инфекции и т.д.).

Цели должны не только отвечать насущным нуждам пациента, но также включать в себя меры по предотвращению заболеваний и реабилитацию.

Существует два вида целей, выделенных для пациентов: краткосрочные и долгосрочные.

Краткосрочными являются цели, которые должны быть выполнены за короткий период времени, обычно меньше, чем за неделю.

Долгосрочными являются цели, которые могут быть достигнуты за более длительный период времени, обычно в течение недель и месяцев (во время пребывания в больнице, при выписке, после выписки). Эти цели обычно направлены на профилактику осложнений, реабилитацию, приобретение знаний о здоровье.

Вернемся к карте сестринского процесса.

Для решения настоящей проблемы № 1 - "чувство распирания в области живота из-за длительной задержки стула" определены две цели:

краткосрочная цель - пациент освободит кишечник в день госпитализации с помощью клизмы, поставленной медсестрой;

долгосрочная цель - самостоятельное опорожнение кишечника пациентом к моменту выписки.

Две другие цели:

краткосрочная цель - в течение недели пациент получит информацию о рациональном питании в результате бесед с медсестрой;

долгосрочная цель - к моменту выписки пациент овладеет комплексом ЛФК и самомассажа в результате постоянных занятий с инструктором ЛФК, что позволяет решить все острые проблемы пациента.

При написании целей должны быть указаны следующие обязательные моменты:

    Мероприятие действие . Например, пациент освободит кишечник, самостоятельно
    опорожнит кишечник, получит информацию, овладеет комплексом ЛФК и самомассажа.

    Критерий - число, время, расстояние. Например, в день госпитализации, к моменту
    выписки, в течение недели, к моменту выписки.

    Условие - помощник, ассистент и т.д.

Например, с помощью клизмы, поставленной медсестрой; самостоятельно; в результате занятий с инструктором ЛФК.

В конечном счете, цель ведет к определению ожидаемых результатов.

Ожидаемые результаты .

Ожидаемый результат является особым, ступенчатым понятием, который ведет к достижению цели. Результатом является изменение поведения пациента как реакции на сестринский уход. Под результатами подразумевают изменения состояния пациента с точки зрения физиологии, социологии, эмоционального и духовного состояния. Данное изменение выявляется через наблюдение за реакцией пациента.

Спланированные до планирования действий медсестры, О.Р. задают направление сестринской деятельности.

О.Р. вытекают из кратко- и долгосрочных целей, сосредоточенных на пациенте, и основываются на сестринских диагнозах. При написании О.Р. сестра должна удостовериться, что результат обозначен в соизмерении с нормами поведения. Они должны быть составлены последовательно, с учетом времени. Это будет способствовать установлению порядка сестринских вмешательств, а также времени разрешения проблемы.

Разнообразие О.Р. определяются для каждой цели и каждого сестринского диагноза. Причиной выделения разнообразных ожидаемых результатов является возможность разрешения нескольких проблем пациента при помощи одного сестринского действия.

(см. карту сестринского процесса)

О.Р. определяются, когда цели, сосредоточенные на пациенте, достигаются. Сестра использует О.Р. в качестве критерия оценки эффективности деятельности сестры.

Чтобы избежать типичных ошибок при написании целей и ожидаемых результатов, необходимо соблюдать основные правила:

1. ЦиР должны быть сосредоточены на пациенте и его поведении и реакции, а не на сестринском вмешательстве .

Правильным будет определить ЦиР так: "пациент освободит кишечник в день госпитализации с помощью клизмы, поставленной медсестрой".

Неверно же определять ЦиР так: "облегчить состояние пациента с помощью клизмы".

2. ЦиР должны быть поставлены так, чтобы их можно было оценить: пронаблюдать, измерить.

3. ЦиР должны быть реальны, так как каждая достигнутая цель вселяет в пациента уверенность в своё скорое выздоровление. Для этого медсестре нужно знать ресурсы здравоохранения, семьи, пациента.

3. Выбор мер по уходу за больными

Это определение объема и методов сестринской помощи (сестринского вмешательства). Существуют 3 категории сестринского вмешательства. Выбор категории основывается на нуждах пациента. Один пациент может иметь все три категории в плане по уходу, в то время как другой пациент может иметь только независимую или взаимозависимую категории в плане по уходу.

1. Независимое вмешательство. Данное вмешательство не требует наблюдения или направления со стороны. Например, вмешательства с целью повышения знаний пациента об адекватном питании или ежедневной деятельности, относящейся к гигиене, массаж, расслабляющая терапия, является самостоятельным действием сестры.

Независимые вмешательства могут разрешить проблемы пациента без консультаций или сотрудничества с врачами или другими мед. работниками. Они не требуют и указаний врача и других специалистов.

2. Взаимозависимое вмешательство. Данные вмешательства производятся сестрой с другим работником здравоохранения. Примером может являться применение гиперинтенсивного лечения, при котором сестра обладает критериями, по которым можно изменять лекарственную и диетическую терапию.

Это сотрудничество можно определить как товарищество, при котором значение двух сторон одинаково ценится обеими сторонами, также признаются и принимаются общие и отдельные сферы деятельности и ответственности, взаимное соблюдение интересов обоих сторон и также целей, которые признаны обеими сторонами.

3. Зависимое вмешательство. Данные вмешательства основаны на инструкции или письменном указании. Управление лечением, использование процедур, смена повязок и подготовка пациента к диагностическим исследованиям являются зависимыми сестринскими вмешательствами.

Предписание различных лечений не входит в область сестринской практики, но сестра отвечает за выполнение предписанного лечения.

Каждое зависимое вмешательство требует ответственности и точного знания. При управлении лечением сестра должна знать классификацию лекарств, их действие, дозировку, побочные эффекты, сестринские вмешательства, относящиеся к их воздействию и побочным эффектам.

При применении процедур или смене повязок сестра должна быть уверена, когда применение процедур необходимо (показания), обладать навыками, необходимыми для их выполнения, предвидеть ожидаемый результат и возможные побочные эффекты.

При назначении диагностического исследования сестра должна спланировать его проведение, подготовить пациента, выявить сестринское применение.

Все вмешательства требуют от сестры оценки и принятия решения. Когда ставится вопрос о выполнении сестринского вмешательства, сестра не должна автоматически выполнять его, она должна определить, является ли предписание необходимым для пациента. Каждая сестра время от времени сталкивается с ненужными и неправильными назначениями. Сестра с хорошей базой знаний узнает ошибку и найдет объяснение, т.к. ошибка может возникнуть при написании указания или при его отражении в карточке пациента. Прояснение указания входит в компетенцию сестры. Сестра, выполняющая неверное или ненужное предписание, также заблуждается, как и тот, кто его написал, и также ответственна за последствия ошибки.

На примере карты сестринского процесса попытаемся определить, какие категории вмешательств имеет план по уходу.

К независимым факторам можно отнести:

1. Двигательная активизация пациента (если профессиональная сестра обладает необходимыми знаниями);

2. Разъяснение принципов рационального питания.
К зависимым факторам относятся:

    Обеспечение диетического питания

    Постановка клизм, применение физиопроцедур

    Введение в рацион фитопрепаратов

    Назначение лекарственных препаратов

Планирование сестринского ухода включает познавательный и письменный процессы. Индивидуальный план ухода является результатом знаний и исследований медсестры так же, как знаний и исследований, приобретенных у консультантов.

План сестринского ухода - письменное руководство по уходу за пациентом. Он отражает проблемы здоровья пациента, которые определяются через обследование, сестринские диагнозы, приоритеты, цели и ожидаемые результаты, сформированные в процессе планирования.

Написание плана позволяет:

1. Уменьшить риск неверного ухода.

В больницах и других лечебных учреждениях США пациент часто получает уход от более, чем одной сестры, врача, постороннего специалиста. Письменный план ухода дает возможность координировать план, проводить консультации и планировать диагностические исследования.

    Дает возможность другой сестре продолжить уход, так как мероприятия плана могут
    выполняться в течении дня или день за днем.

    Медсестрам обмениваться информацией.

Сестры составляют свои отчеты на основе сестринского ухода и лечения, предусмотренного планом ухода. После обмена информацией сестры обсуждают планы ухода за пациентом с теми, кто продолжит уход. Таким образом, все сестры способны обсудить текущую и уже хорошо изученную информацию о плане ухода за пациентом.

4. Проводить реабилитацию после выписки.

Письменный план ухода за пациентом предусматривает и нужды пациента после выписки его из больницы. Это особенно важно для пациента, так как он пройдет через длительный курс реабилитации в обществе (после хирургических вмешательств и т.д.).

Результатом полного и точного планирования сестринского ухода является индивидуализация, координация и продолжение сестринского ухода. Планирование устанавливает рамки сестринского ухода, которые должны быть соблюдены

IV этап сестринского процесса -выполнение плана сестринского ухода

Сделать всё необходимое для выполнения плана ухода за пациентом (идентично общей цели сестринского процесса).


Выполнение или реализация - это мероприятия, направленные на:

    Помощь при болезни.

    Профилактику заболеваний и осложнений.

    Укрепление здоровья.

По теории, выполнение плана сестринского ухода следует за планированием, однако на практике выполнение может начаться сразу после обследования.

Прибегать к немедленному выполнению необходимо иногда, когда есть угроза физическому, психологическому и духовному состоянию пациента.

Выполнение является категорией сестринского поведения, в котором действия, необходимые для достижения ожидаемого результата сестринского ухода, производят вплоть до завершения.

    оказание помощи

    управление деятельностью в повседневной жизни

    обучение и консультирование пациента и его семьи

    оказание непосредственной помощи в интересах пациента

    оценка работы медперсонала

    запись и обмен информацией


Таблица № 7

После того, как план ухода уже выработан и определен I этап сестринской помощи, медсестра приступает к выполнению, т.е. проводит то или иное сестринское вмешательство.

Сестринское вмешательство - это любое действие м/с, которое осуществляет план сестринского ухода или любую задачу этого плана. Сестринская помощь может быть зависимой, независимой, взаимозависимой (см. III ступень). Помимо этого, сестринские вмешательства могут полностью основываться на протоколах и указаниях.

Протоколом является письменный план, точно определяющий процедуры, проводимые во время обследования.

Указание - это документ, содержащий правила, процедуры, устав для проведения ухода за пациентом. Указания одобрены и подписаны лечащим врачом до их применения. Обычно они применяются в отделениях реанимации, где нужды пациента могут быстро изменяться и требуют особого внимания. Указания также применяются в лечебных учреждениях, где нет возможности сразу посоветоваться с врачом.

Указания и протоколы дают сестре легальную защищенность вмешиваться в интересах пациента. Ответственность медсестры одинаково велика для всех типов вмешательства.

Методы выполнения

Существуют различные методы ухода за больными. Чтобы достичь поставленных целей, медсестра делает выбор из следующих методов:

    Помощь в выполнении деятельности, связанной с жизненными потребностями.

    Советы и инструкции больному и его семье.

    Уход за больными для достижения терапевтических целей.

    Уход за больными для облегчения достижения целей лечения больных.

5. Наблюдение и оценка работы других штатных сотрудников.

Для достижения целей ухода за больными, независимо от применяемых методов, м/сдолжна обладать теоретическими знаниями, практическими навыками и навыками общения с пациентом и его родственниками.

Что же конкретно предусматривает каждый из этих методов?

1. Помощь в выполнении деятельности, связанной с жизнен н ыми потребностями.

Это деятельность , связанная с ежедневными нуждами, обычно проводится в течение дня и включает еду, одевание, умывание, подачу судна и т.д.

Потребность пациента в помощи может быть: временной, постоянной и реабилитационной.

В случаях временного ухода - такая помощь требуется в течение короткого времени, например, при переломе верхних конечностей больной будет нуждаться в помощи до снятия гипса.

У пациента, не имеющего возможности к самообслуживанию из-за поражения шейного отдела позвоночника, потребность в помощи будет постоянной.

Реабилитация поможет пациенту приобрести новые навыки выполнения ежедневных потребностей, чтобы стать более независимым и способным к самоуходу.

2. Советы и инструкции больному и его семье

Совет - это эмоциональная, интеллектуальная и психологическая помощь. Совет, в качестве метода выполнения, помогает пациенту приспособиться к новым условиям жизни, справиться с проблемами, стрессами и облегчает межличностные отношения между пациентами, семьей и медперсоналом. Совет очень близко связан с обучением. Обучение (инструкции), в качестве метода выполнения, используется для информирования пациентов о состоянии их здоровья, для приобретения больными необходимых навыков самоухода. Медсестра несет ответственность за определение необходимости в обучении пациентов и за качество даваемых инструкций.

3. Уход за больным для достижения терапевтических целей

Для достижения целей лечения м/с предпринимает вмешательства для:

а) спасения жизни пациента (реанимационные мероприятия, сдерживание буйного
пациента и т.д.);

б) компенсации неблагоприятных реакций, вызванных процедурами, медикаментами, диагностическими исследованиями.

Например, у пациента ранее была непереносимость или аллергическая реакция на введение витаминных препаратов. В этом случае м/с должна:

    прекратить введение лекарственных средств;

    записать симптомы, если они есть;

    сообщить врачу, по его назначению, ввести антигистаминные препараты.

в) профилактические мероприятия.

Они направлены на предотвращение осложнений или обострений заболевания. Например, профилактические мероприятия при выявлении аллергической реакции:

    отметить в истории болезни непереносимость витаминных препаратов;

    известить пациента и его семью;

    дать совет пациенту, что он должен делать при повторном назначении этих лекарственных веществ.

4. Уход за больным для облегчения достижения целей лечения больного

Это - мероприятия, направление на создание благоприятной окружающей среды для пациента, т.е. соблюдение лечебно-охранительного режима.

Самым ранним этапом создания нужной среды будет, например, при поступления пациента в больницу его необходимо:

    проводить в палату;

    познакомить с обслуживающим персоналом и др. пациентами;

    познакомить с распорядком дня и устройством отделения;

    обеспечить уединенность для выполнения гигиенических нужд и т.д.;

Поощрять и одобрять малейшие усилия пациента, направленные на выздоровление.
Уход за больным и другие терапевтические меры предназначены для нужд пациента,

планы по уходу должны быть гибкими, что дает возможность пациенту иметь выбор.

5. Наблюдение и оценка работы других членов персонала

Медсестра, разрабатывающая план ухода, часто не выполняет все вмешательства сама. Часть их поручается другим сотрудникам (младшие медсёстры, помощник медсестры и т.д.). Но ответственность за качество выполненных мероприятий несет медицинская сестра.

V этап сестринского процесса – оценка достижения целей

и ожидаемых результатов

Цель - определить, в какой степени достигнуты результаты.


Оценка – это заключительный этап сестринского процесса, который предусматривает три различных аспекта:

    Оценка реакций пациента на вмешательство. Мнение пациента о вмешательстве.

    Оценка достижения поставленных целей.

    Оценка качества оказанной помощи. Влияние вмешательства на пациента.

Оценка проводится постоянно, когда медсестра общается с пациентом. Основное внимание уделяется улучшению состояния пациента.

Что же предусматривает каждый аспект этой ступени сестринского процесса?

    Оценка реакций пациента на сестринские вмешательства.

Мнение пациента о вмешательстве.

Ухаживая за пациентом, медсестра сравнивает достигнутые результаты. Например, уменьшение болевых симптомов, улучшение знаний о своем заболевании и т.д.

Сравнение проводится вместе с больным, и результаты основываются на его мнении.

2. Оценка достижения поставленных целей.

Сестринский уход нужен для того, чтобы помочь пациенту решать его проблемы со здоровьем, предотвращать появление потенциальных проблем и поддерживать его здоровье. Оценка показывает, была ли достигнута цель.

Например, во время обследования пациент чувствует сильную боль в животе, держится руками за живот, на лице - гримаса боли. Это основные показания сестра использует для определения сестринского диагноза, ставит цели, планирует уход, проводит вмешательства. После того, как сестринские действия были произведены, сестра переоценивает состояние пациента, наблюдая за реакцией пациента. Для объективной оценки степени успеха в достижении целей медсестра должна проделать следующее:

    Проверить поставленную цель для определения точных желаний пациента относительно его

Поведения или реакции.

    Оценить наличие у пациента этого поведения или реакции.

    Сравнить целевые критерии с поведением или реакцией.

    Определить степень согласованности между целевыми критериями и поведением или

Реакцией.

Таблица № 8

3. Оценка качества сестринской помощи.

Этот аспект оценки заключается в измерении качества сестринского ухода.

Оценочные критерии - это просто навыки оценки и техника, используемая для сбора данных для оценки. Эта оценка складывается из мнения пациента или его реакции на качество предоставленной помощи и наличие осложнений для проведения вмешательства.

Оценка считается положительной, когда цели и ожидаемые результаты достигнуты, отрицательной, если результаты нежелательны, или не удалось избежать потенциальных проблем. В этом случае сестра должна изменить план по уходу, и сестринский процесс согласовывается вновь. Это согласование продолжается до решения всех проблем.

Рассмотрим варианты достижения целей на следующих примерах. См. приложение № 1 и карту сестринского процесса.

Удостоверившись, что ожидаемые результаты и цели были достигнуты, медсестра обращается с этими оценками к пациенту, если он соглашается, то медсестра прерывает эту ветвь плана по уходу. Если цели не достигнуты или достигаются частично, необходимо установить факторы, мешающие достижению целей, и устранить их.

Когда время госпитализации заканчивается, многие пациенты выписываются до того, как были достигнуты все цели, и все проблемы были решены.

Значение сестринского процесса

    Сестринский процесс повышает качество ухода;

    Поддерживает связь между медицинским персоналом;

    Стимулирует медицинских сестёр к повышению уровня профессиональной подготовки;

    Осуществляет постоянное наблюдение за пациентом;

    Медицинский персонал относится к пациенту как к личности;

    Благодаря сестринской истории болезни легче оценить качество работы медсестры, её компетентность;

    Пациент, медсестра, окружение становятся участниками сестринского процесса.

Психологические аспекты общения с пациентом

ПРИМЕРНЫЙ АЛГОРИТМ ИНТЕРВЬЮ

(беседы)

1. Приветствие. Ключ к общению - приветствие. В больничной ситуации приемлемы как основные формы "Здравствуйте", так и родственные формы "Доброе утро!", "Добрый день!", "Приветствую Вас! ", "Рад Вас приветствовать! ".

Адресат приветствия - наш пациент, поэтому исключаются формы фамильярности, непринужденные ("Привет!") и церемонные, шутливые. В обращении к пациенту речевая форма "Здравствуйте" должна содержать правильную тональность общения; знаки доброжелательности служат базой (ключом) и необходимым контактом.

    Представьтесь: "Меня зовут...".

    Выясните, настроен ли пациент с Вами беседовать. С этой целью можно задать следующие вопросы: "Вы позволите с Вами побеседовать?" или "Можно мне задать Вам несколько вопросов?".

4 . Попросите пациента представиться, недопустима в общении фамильярность (обращение на "ты", по имени и т.д.). Это может расцениваться пациентом с обидой. Обращение на "Вы" свидетельствует, о большой вежливости. Оправдана деликатно-равноправная форма обращения на "Вы" и по имени отчеству.

    Спросите о его жалобах, о том, когда появились нарушения, когда впервые пациент обратил на них внимание. "Как Вы себя чувствуйте?", "Что Вас беспокоит?", или "На что Вы жалуетесь?".

    Выработка адекватной самооценки пациентом его тревог и беспокойства. Выясните, какое значение имеет данный симптом, как пациент реагирует на свои жалобы; и постарайтесь позитивно истолковать его состояние. Пациент будет испытывать облегчение, если медсестра рассеет его страхи.

Например. У пациента недавно возникла стенокардия. Пациент проявляет тревожность по поводу хода заболевания. В таком случае, можно рассказать ему о фактах риска и в зависимости от конкретных обстоятельств, сказать: "Вы не курите, у Вас нет диабета, давление у Вас сейчас нормальное, все это благоприятные факторы. Вы заболели недавно, значит, болезнь еще не запущена".

ТАКИЕ БЕСЕДЫ НЕ ПРОСТО УСПОКАИВАЮТ ПАЦИЕНТА, ОНИ ПОКАЗЫВАЮТ ЕМУ БЛАГОПРИЯТНУЮ ПЕРСПЕКТИВУ И НАСТРАИВАЮТ НА ОПТИМИСТИЧЕСКИЙ ЛАД.

    Лечение до настоящего времени и его результаты.

    Предполагаемые причины заболевания.

    Время возникновения первого симптома.

    Перенесенные заболевания (операции, травмы, аллергии, ранения).

    Факторы риска, привычки (кофе, курение, алкоголь, наркотики).

    Заболевание членов семьи, семейный анамнез (факторы риска в плане раковых заболеваний, сердечнососудистых заболеваний, диабета, болезни почек, гипертонии, психических расстройств).

    Условия труда, профессиональные вредности, среда обитания (неблагоприятная экологическая обстановка).

    Психологический климат (круг общения, темперамент, характер, уровень развития в целом, образ жизни, верования, моральные ценности).

    Социальный статус (роль в семье, на работе, финансовое положение).

    Влияние заболевания и проблем пациента на него и его окружение. (Ощущает ли он в связи с этим тревогу или внутреннее напряжение):

а) на профессиональную деятельность;

б) на семью или партнера;

в) на межличностные отношения, на контакты;

г) на дальнейшие перспективы.

Приложение № 1

    Вы должны быть уверены, что Ваша беседа будет проходить в тихой неофициальной обстановке без отвлечений и не будет прерываться.

    Используйте наиболее надежный источник информации - если не самого пациента, то его ближайших родственников.

    Используйте полученные ранее Вами сведения о диагнозах пациента (если они Вам известны), чтобы заранее спланировать, на какой информации сосредоточить внимание и получить необходимые нам факты.

    Прежде чем начать, объясните, что, чем больше Вы будете знать о пациенте и его семье, тем лучший уход Вы сможете ему обеспечить, вот почему Вы задаете ему много вопросов.

    Во время интервью делайте короткие записи. Аккуратно записывайте даты, число и длительность госпитализации и начала заболевания.

НЕ РАССЧИТЫВАЙТЕ НА СВОЮ ПАМЯТЬ!

    Не пытайтесь вести записи в форме законченных предложений.

    Будьте спокойны, неторопливы и проявляйте сочувствие. Проявляйте искренний интерес и сочувствие. (Чуткость ободряет пациента и облегчает выражение его чувств).

    Не проявляйте досады и раздражения, если у пациента возникает провал памяти. Если отнесетесь к этому с пониманием, он может вспомнить необходимую информацию позже при ответе на соответствующий вопрос.

    Используйте должным образом визуальный контакт. Внимательно наблюдайте за "языком тела" пациента.

    Не останавливайте подолгу взгляд на пациенте или на своих записях.

    Используйте нейтральные вопросы, способствующие формулированию пациентом своих ощущений и сообщению дополнительной информации.

    В меру обоснованно используйте наводящие вопросы с тем, чтобы сосредоточиться на неясных моментах. Используйте соответствующие слова пациента для уточнения информации. Говоря "режущая боль", Вы имеете в виду внезапную, сильную боль?"

    Используйте понятную пациенту терминологию . Если Вы сомневаетесь в том, что он Вас понимает, спросите его о том, что он вкладывает в то или иное понятие.

14. Чтобы пациент почувствовал целесообразность интервьюирования, прежде всего, спросите о его жалобах. НЕ НАЧИНАЙТЕ С ЛИЧНЫХ ВОПРОСОВ!

    Позвольте пациенту закончить фразу, даже если он чересчур многословен. Только потом задайте вопросы. Не перескакивайте с темы на тему. Не повторяйте вопрос без необходимости. Если необходимо повторить вопрос, перефразируйте его для лучшего понимания.

    Отнеситесь с пониманием к тому, что говорит пациент. Простой кивок, поддакивание, одобрительный взгляд помогут ему продолжить рассказ, особенно если пациент не доминантный.

    Называйте пациента по имени. Проявляйте дружелюбие, участие и заботу.

    Не утрачивайте профессионализма. Говорите ясно, медленно и отчетливо.

    УМЕЙТЕ СЛУШАТЬ !

Вообще, замечено, что уравновешенный человек с чувством собственного достоинства подходит к собеседнику ближе, тогда как беспокойные, нервные люди стараются держаться подальше, особенно от собеседника противоположного пола. Когда неизвестно, в каком положении пациент чувствует себя наиболее удобно, то надо понаблюдать, как он входит в кабинет, в палату, как сидит, стоит, берег стул, как движется, когда думает, что па него смотрят. Важно обращать внимание на взаимное расположение и позы медсестры и пациента.

II . Позы - отражают состояние человека и соотношение к происходящему. Почти у каждого человека есть своя излюбленная поза или позы, поэтому не всегда легко понять, является ли данная поза выражением состояния человека в данный момент или это всего лишь дань привычке. С другой стороны, частое предпочтение той или иной позе может выражать подверженность человека соответствующему состоянию. Вместе с тем, если человек часто принимает одну и ту же позу, как бы по привычке, не исключено, что он чаще всего задумчив, неконтактен и т.п.

Позы могут быть открытыми и закрытыми. Открытая поза определяется: поворотом корпуса и головы к собеседнику, раскрытостью ладоней, нескрещенным положением ног, расслабленностью мышц, "прямым" взглядом в лицо.

Закрытая поза: скрещенные ноги или руки обычно отражают защитную реакцию и нежелание общаться.

Быстрый резкий наклон или поворот головы, жестикуляция указывают на то, что пациент хочет высказаться.

Ш. Движение и жесты.

Под движением понимают перемещение в пространстве всего тела, а под жестами - движения различных частей тела, по основное значение имеют движения головы, плеч и рук.

    Коммуникативные жесты (жесты, которые имеют самостоятельное значение и не нуждаются в словесных пояснениях - кивок головы в знак согласия, поднятый палец) производятся, как правило, специально (сознательно), чтобы сообщить собеседнику нужную информацию. Это - жесты: приветствия, прощания, вопросительные, утвердительные, угрожающие, отрицающие и т.д.

    Выразительные жесты и движения часто бывают непроизвольными. По ним можно "прочитать" состояние человека, его отношение к происходящему.А также определить оценку людей, событий и т.п., которые возможно, он хотел бы скрыть. Жесты: незнания, недоверия, растерянности, удивления, иронии, неудовольствия, страдания, одобрения, радости, восторга.

Коммуникативные и выразительные жесты могут не совпадать с речью и даже противоречить ей. При этом возможны два варианта:

    Человек намеренно хочет жестом выразить нечто совсем иное, чем то, что формирует в
    речи (подтекст).

    Человек говорит не то, что чувствует, и жесты выдают, но для лучшего понимания пациента надо уметь их "читать".

Активная жестикуляция часто отражает положительные эмоции и воспринимается как признак заинтересованности и дружелюбия. Чрезмерное жестикулирование, однако, может быть выражением беспокойства и неуверенности. Большинство жестов являются многовариантными. Например, взмах руки можно употреблять как знак отчаяния, привлечения внимания или отказа от чего-либо. Кивки головой не всегда означают согласие - часто они лишь показывают говорящему, что его слушают и готовы слушать дальше. Они как бы дают разрешение говорящему продолжать речь.

IV . Мимика. Эко координирование движения мышц лица, отражающие состояния, чувства, эмоции. "Мимика - это зрительный язык", главное средство словесного общения, это сигнал о намерениях человека, ею эмоциях. Различают мимику верхней и нижней части лица- Замечено, что верхняя часть лица контролируется нами больше, чем нижняя. Поэтому, если хочешь больше узнать о человеке, его состоянии, мотивах и даже мыслях, смотри чаще на рог, крылья носа, подбородок. Особенно экспрессивны губы человека. Плотно сжатые губы выражают глубокую задумчивость, изогнутые - сомнение или сарказм. Уголки рта - показатели жизненного тонуса личности. Уголки рта вниз - симптом подавленности, депрессивного состояния, при радостном, бодром состоянии уголки рта выравниваются. Лобная мышца - это мышца внимания или настороженности, она включается при опасности, агрессивности.

Страх: брови почти прямые и кажутся несколько приподнятыми, глаза расширены, нижнее веко напряжено, а верхнее слегка приподнято. Po т при страхе открыт и имеет узкую эллиптическую форму, губы напряжены и немного растянуты. Чем сильнее страх, тем больше оттянуты назад уголки рта.

Глаза. Контакт глаз.

Люди общительные, открытые, ориентированные на других (экстраверты), смотрят на собеседника пристальнее и дольше, чем интроверты, люди замкнутые, направленные на себя. Взгляд выражает нашу заинтересованность, помогает нам сосредоточить внимание на том, что нам говорят. Если говорящий, то смотрит в глаза, то отводит их в сторону - это значит, что он еще не закончил говорить. Если смотрит прямо в глаза по завершению фразы - сообщает, что он все сказал. Взгляд не должен быть слишком пристальным (прямо в зрачки). Пристальный взгляд может восприниматься как признак враждебности, поэтому в ситуациях спора люди избегают визуального контакта, чтобы этот контакт не был понят как выражение враждебности.

Сестринский процесс состоит из пяти основных этапов. ПЕРВЫЙ ЭТАП – обследование пациента с целью сбора информации о состоянии здоровья. Цель обследования – собрать, обосновать и взаимосвязать полученную информацию о пациенте для того, чтобы создать информационную базу данных о нем, о его состоянии в момент обращения за помощью. Главная роль в обследовании принадлежит расспросу. Собранные данные записываются в сестринскую историю болезни по определенной форме. Сестринская история болезни – юридический протокол-документ самостоятельной, профессиональной деятельности медсестры в рамках ее компетенции. ВТОРОЙ ЭТАП – установление проблем пациента и формулировка сестринского диагноза. Проблемы пациента подразделяются на: основные или настоящие, сопутствующие и потенциальные. Основные проблемы – это проблемы, которые беспокоят пациента в настоящий момент. Потенциальные проблемы – те, которые еще не существуют, но могут появиться с течением времени. Сопутствующие проблемы – это не экстремальные и не опасные для жизни потребности и не имеют прямого отношения к заболеванию или прогнозу. Таким образом, задача сестринской диагностики – установить все настоящие или возможные в будущем отклонения от комфортного, гармоничного состояния, установить то, что наиболее тяготит пациента в настоящий момент, является для него главным и попытаться в пределах своей компетенции скорректировать эти отклонения. Медсестра рассматривает не заболевание, а реакцию пациента на болезнь и свое состояние. Эта реакция может быть: физиологической, психологической, социальной, духовной. ТРЕТИЙ ЭТАП – планирование сестринской помощи. План ухода Постановка целей: Участие пациента Стандарты сестринской 1. Краткосрочные и его семьи практики 2. Долгосрочные ЧЕТВЕРТЫЙ ЭТАП – осуществление плана сестринских вмешательств. Сестринские вмешательства Категории: Потребность пациента Методы ухода: в помощи: 1. Независимая 1. Временная 1. Достижение терапевтических 2. Зависимая 2. Постоянная целей 3. Взаимозависимая 3. Реабилитирующая 2. Обеспечение ежедневных жизненных потребностей и т.п. ПЯТЫЙ ЭТАП – оценка эффективности сестринского процесса. Эффективность сестринского процесса Оценка действий Мнение пациента Оценка действий медсестры медицинской сестры или его семьи руководителем (старшей и главной (самолично) медицинскими сестрами) Оценка всего сестринского процесса производится в том случае, если пациент выписывается, если его перевели в другое лечебное учреждение, если пациент умер или в случае длительного заболевания. Осуществление и внедрение сестринского процесса в ЛПУ поможет решить следующие задачи: Повысить качество и снизить сроки лечебного процесса без привлечения дополнительных средств; Снизить потребность во врачебных кадрах, создавая «сестринские отделения, дома, Hospis» с минимальным количеством врачей; Повысить роль медсестры в лечебном процессе, что немаловажно для достижения более высокого социального статуса медицинской сестры в обществе; Внедрение многоуровневого сестринского образования позволит обеспечить лечебный процесс кадрами с дифференцированным уровнем подготовки.

1. Сестринское обследование .

2. Сестринская диагностика.

3. Планирование сестринского вмешательства.

4. Реализация сестринского плана (сестринское вмешательство).

5. Оценка результата.

Этапы последовательны и взаимосвязаны.

1 этап СП - сестринское обследование .

Это сбор информации о состоянии здоровья пациента, его личности, образе жизни и отражение полученных данных в сестринской истории болезни.

Цель : создание информативной базы о пациенте.

Фундамент сестринского обследования составляет учение об основных жизненно важных потребностях человека.

Потребность есть физиологический и (или) психологиче­ский дефицит того, что существенно для здоровья и благополу­чия человека.

В сестринской практике используется классификация потреб­ностей Вирджинии Хендерсон (Модель сестринского дела В. Хендерсон, 1966 г.) , которая всё их многообразие свела к 14-ти наиболее важным и назвала их видами повседневной деятельности. В своей работе В. Хендерсон использовала теорию иерархии потребностей А. Маслоу (1943г.). По его теории, одни потребности для человека более существенны, чем др. Это позволило А. Маслоу классифицировать их по иерархической системе: от физиологических (низший уровень) до потребностей в самовыражении (высший уровень). А. Маслоу изобразил эти уровни потребностей в виде пирамиды, поскольку именно эта фигура имеет широкое основание (основу, фундамент), как и физиологические потребности человека, являются основой его жизнедеятельности (учебник стр. 78):

1. Физиологические потребности.

2. Безопасность.

3. Социальные потребности (общение).

4. Самоуважение и уважение.

5. Самовыражение.

Прежде чем думать об удовлетворении потребностей высшего уровня, необходимо удовлетворить потребности низшего порядка.

Учитывая реалии российского практического здравоохранения, отечественные исследователи С.А. Мухина и И.И. Тарновская предлагают осуществлять сестринскую помощь в рамках 10 фун­даментальных потребностей человека:


1. Нормальное дыхание.

3. Физиологические отправления.

4. Движение.

6. Личная гигиена и смена одежды.

7. Поддержание нормальной температуры тела.

8. Поддержание безопасности окружающей среды.

9. Общение.

10. Труд и отдых.


Основные источники информации о пациенте


пациент члены семьи, обзор

мед. персонал мед. документация данные спец. и мед.

друзья, обследований лит-ры

прохожие

Методы сбора информации о пациента


Таким образом, м/с оценивает следующие группы параметров: физиологические, социальные, психологические, духовные.

субъективную – включает чувства, эмоции, ощущения (жалобы) самого пациента относительно своего здоровья;

М/с получает два вида информации:

объективную – данные, которые получены в результате наблюдений и обследований, проводимых медсестры.

Следовательно, источники информации также разделяются на объективные и субъективные.

Сестринское обследование является независимым, и не может подменяться врачебным, т. к. задача врачебного обследования - назначить лечение, в то время как сестринского - предоставить мотивированный индивидуализированный уход.

Собранные данные записываются в сестринскую историю болезни по определённой форме.

Сестринская история болезни - юридический протокол документ самостоятельной, профессиональной деятельности медсестры в рамках её компетенции.

Цель сестринской истории болезни - контроль за деятельностью медсестры, выполнением ею плана ухода и рекомендаций врача, анализ качества оказания сестринской помощи и оценка профессионализма медсестры.

2 этап СП – сестринская диагностика

- это клиническое суждение медсестры, в котором дается описание характера существующей или потенциальной ответной реакции пациента на болезнь и свое состояние, с желательным указанием вероятной причины такой реакции.

Цель сестринской диагностики : проанализировать результаты обследования и определить, с какой проблемой здоровья сталкивается пациент и его семья, а так же определить направление сестринского ухода.

С точки зрения медицинской сестры, проблемы появляются то­гда, когда у пациента, в силу определенных причин (болезнь, трав­ма, возраст, неблагоприятная обстановка), возникают следующие затруднения:

1. Не может самостоятельно удовлетворить какие-либо из потребностей или у него возникают трудности в их удов­летворении (например, не может принимать пищу из-за болей при глотании, не может передвигаться без дополни­тельной опоры).

2. Пациент удовлетворяет свои потребности самостоятельно, но то, как он их удовлетворяет, не способствует сохранению его здоровья на оптимальном уровне (например, при­страстие к жирной и пряной пище чревато заболеванием пищеварительной системы).

Проблемы м.б. :

Существующие и потенциальные.

Существующие – это проблемы, которые беспокоят пациента в настоящий момент.

Потенциальные – те, которые не существуют, но могут появиться с течением времени.

По приоритетности проблемы классифицируются как первичные, промежуточные и вторичные (приоритеты, следовательно, классифицируются аналогично).

К первичным относят проблемы, связанные с повышенным риском и требующие экстренной помощи.

Промежуточные не представляют серьезной опасности и допускают отсрочку сестринского вмешательства.

Вторичные проблемы не имеют прямого отношения к заболеванию и его прогнозу.

На основании выявленных проблем пациента медсестра приступает к постановке диагноза.

Отличительные особенности сестринского и врачебного диагнозов:

Врачебный диагноз сестринский диагноз

1. выявляет конкретное заболевание выявляет ответную реакцию пациента

или сущность патологического на болезнь или свое состояние

процесса

2. отражает врачебную цель – вылечить сестринскую – решение проблем

пациента при острой патологии пациента

или вывести заболевание в стадию

ремиссии при хронической

3. как правило, правильно поставленный меняется периодически

врачебный диагноз не меняется

Структура сестринского диагноза:

1 часть – описание ответной реакции пациента на болезнь;

2 часть – описание возможной причины такой реакции.

Например : 1ч. – нарушение в питании,

2ч. – связанное с низкими финансовыми возможностями.

Классификация сестринских диагнозов (по характеру реакции пациента на болезнь и своё состояние).

Физиологические (например, пациент не удерживает мочу при напряжении). Психологические (например, пациент боится не проснуться после наркоза).

Духовные - проблемы высшего порядка, связанные с пред­ставлениями человека о его жизненных ценностях, с его вероисповеданием, поисками смысла жизни и смерти (одиночество, чувство вины, страх смерти, потребность в святом причастии).

Социальные - социальная изоляция, конфликтная ситуация в семье, финансовые или бытовые проблемы, связан­ные с выходом на инвалидность, переменой места житель­ства и т.д.

Таким образом, в модели В. Хендерсон сестринский диагноз всегда отражает дефицит самоухо­да, имеющийся у пациента, и направлен на его замещение и пре­одоление. Как правило, у пациента одновременно диагностируется не­сколько проблем, связанных со здоровьем. Проблемы пациента учитываются одновременно: сестра решает все проблемы, которые ставит, в порядке их значимости, начи­ная с самых важных и далее по порядку. Критерии выбора порядка значимости проблем пациента:

Главное, по мнению самого пациента, самое тягостное и па­губное для него либо препятствующее осуществлению самоухода;

Проблемы, способствующие ухудшению течения заболевания и высокому риску развития осложнений.

3 этап СП – планирование сестринского вмешательства

Это определение целей и составление ин­дивидуального плана сестринского вмешательства отдельно для каждой проблемы пациента, в соответствии с порядком их значи­мости.

Цель : Исходя из потребностей пациента, выделить приоритетные проблемы, разработать стратегию достижения поставленных целей (план), определить критерий их выполнения.

Для каждой приоритетной проблемы записываются конкретные цели ухода, а для каждой конкретной цели нужно подобрать конкретное сестринское вмешательство.

Приоритетная проблема – конкретная цель – конкретное сестринское вмешательство

В сестринской практике цель - это ожидаемый конкретный положительный результат сестринского вмешательства по опреде­лённой проблеме пациента.

Требования, предъявляемые к целям:

  1. Цель должна соответствовать поставленной проблеме.
  2. Цель должна быть реальной, достижимой, диагностичной (возможность проверки достижения).
  3. Цель должна быть сформулирована в пределах сестринской, а не врачебной компетенции.
  4. Цель должна быть сосредоточена на пациенте, то есть должна формулироваться «от пациента», отражать то существенное, что получит пациент в результате сестринского вмешательства.
  5. Цели должны быть конкретными , следует избегать рас­плывчатых общих формулировок («пациент будет чувство­вать себя лучше», «у пациента не будет дискомфорта», «па­циент будет адаптирован).
  6. Цели должны иметь конкретные сроки их достижения.
  7. Цель должна быть понятна пациенту, его семье, а также другим медицинским работникам.
  8. Цель должна предусматривать только положительный результат:

Уменьшение или полное исчезновение симптомов, вызывающих страх у пациента или тревогу у сестры;

Улучшение самочувствия;

Расширение возможностей самоухода в рамках фундаментальных потребностей;

Изменение отношения к своему здоровью.

Виды целей

Краткосрочные Долгосрочные

(тактичные) (стратегические).

Структура цели

исполнение критерий условие

(действие) (дата, время, расстояние) (с помощью кого или чего-либо)

Например, пациент пройдет с помощью костылей 7 метров на восьмой день.

Четко сформулированные цели ухода сестринского ухода позволяют м/с составить план по уходу за пациентом.

План - это письменное руководство, предусматривающее последовательность и фазность сестринских вмешательств, необходимых для достижения целей ухода.

Стандарт плана по уходу – базовый уровень сестринского обслуживания, обеспечивающий качественный уход по определенной проблеме пациента, вне зависимости от конкретной клинической ситуации. Стандарты могут быть приняты как на федеральном, так и на местном уровне (департаменты здра­воохранения, конкретное лечебно-профилактическое учреждение). Примером стандарта сестринской практики может служить ОСТ «Протокол ведения больных. Профилактика пролежней».

Индивидуальный план по уходу – письменное руководство по уходу, представляющее собой подробное перечисление действий м/с, необходимых для достижения целей ухода по определенной проблеме пациента, с учетом конкретной клинической ситуации.

Планирование обеспечивает:

· преемственность сестринского ухода (координирует работу сестринской бригады, помогает поддерживать связь с др. специалистами и службами);

· снижение риска некомпетентного ухода (позволяет контролировать объём и правильность оказания сестринской помощи);

· возможность определения экономических затрат.

В конце третьего этапа сестра в обязательном порядке согла­совывает свои действия с пациентом и его семьёй.

4 этап СП – сестринское вмешательство

Цель : Сделать все необходимое для выполнения плана ухода за пациентом.

Центральны пунктом сестринского вмешательства, всегда является дефицит в способности пациента удовлетворять свои потребности.

1. – пациент не может осуществлять самоуход;

2. – пациент может осуществлять самоуход частично;

3. – пациент может осуществлять самоуход полностью.

В связи с этим различны и системы сестринского вмешательства:

1 – полностью компенсирующая система помощи (параличи, бессознательное состояние, запреты пациенту на перемещение, психические нарушения);

2 – частичная система помощи (большинство пациентов, находящихся в стационаре);

3 – консультативно-поддерживающая система (амбулаторное обслуживание).

Типы сестринских вмешательств:

5 этап СП – оценка результата

– это анализ ответных реакций пациента на сестринское вмешательство.

Цель : Определить, в какой степени достигнуты, поставленные цели (анализ качества сестринской помощи)

Процесс оценки включает;

1 – определение достижения цели;

2 – сравнение с ожидаемым результатом;

3 – формулировка выводов;

4 – отметка в сестринской документации эффективности плана ухода.

Реализация каждого пункта плана по уходу за пациентом приводит в общем случае к новому состоянию пациента, которое может быть:

Лучше прежнего состояния

Без изменений

Хуже прежнего

Оценка осуществляется сестрой непрерывно, с определённой периодичностью, которая зависит от состояния пациента и характе­ра проблемы. Например, оценка в отношении одного пациента бу­дет производиться в начале и в конце смены, а в отношении другого - каждый час.

Если поставленные цели достигнуты и проблема решена, м/с должна удостоверить это, расписавшись под соответствующей целью и поставить дату.

К основным критериям эффективности сестринской помощи относятся:

Прогресс в достижении целей;

Ответная положительная реакция пациента на вмешательство;

Соответствие полученного результата ожидаемому.

Если же, цель не достигнута, необходимо:

Выявить причину – поиск допущенной ошибки.

Изменить саму цель, сделать ее более реалистичнее.

Пересмотреть сроки.

Ввести необходимые коррективы в план сестринской помощи

ПРОБЛЕМНЫЕ ВОПРОСЫ:

  1. Как вы понимаете смысл определения: сестринское дело есть способ удовлетворения жизненно важных потребно­стей человека? Приведите примеры связи проблем пациен­та, требующих вмешательства сестры, с нарушением удов­летворения потребностей его организма в ситуации болезни.
  2. Почему сестринский процесс называют круговым и циклическим процессом?
  3. Охарактеризуйте различия традиционного и современного подхода к организации сестринской помощи пациенту.
  4. Правильно ли сформулирована цель сестринского вмеша­тельства: медсестра обеспечит полноценный сон пациен­та? Приведите свой вариант.
  5. Почему сестринскую историю болезни называют зеркалом, отражающим квалификацию и уровень мышления медицин­ской сестры?

Тема: «ВНУТРИБОЛЬНИЧНАЯ ИНФЕКЦИЯ.

ИНФЕКЦИОННАЯ БЕЗОПАСНОСТЬ. ИНФЕКЦИОННЫЙ КОНТРОЛЬ»

План:

· Понятие о ВБИ.

· Основные факторы, способствующие распространенности ВБИ.

· Возбудители ВБИ.

· Источники ВБИ.

· Инфекционный процесс. Цепочка инфекционного процесса.

· Понятие о санитарно-эпидемиологическом режиме и его роли в профилактике ВБИ.

· Приказы МЗ, регламентирующие санитарно-эпидемиологический режим в ЛПУ.

· Понятие о деконтаминации. Уровни обработки рук.

Предварительный просмотр:

Государственное бюджетное образовательное учреждение

среднего профессионального образования

города Москвы

Медицинское училище № 13

Департамента здравоохранения города Москвы

«Третий этап сестринского процесса: определение целей сестринского вмешательства, планирование ухода».

Дисциплина: «Основы сестринского дела»

Специальность: 060109 «Сестринское дело» базовый уровень СПО

2014 год

Утверждено на заседании методического совета

Протокол №

от 20____ года

Рецензенты:

Согласовано с зам. директора по научно-методической работе

« ____» ______________ 20___ г.

Чугунова Н.А.

Одобрено на заседании ЦМК № 1

Протокол №

«_____» _____________ 20____ г.

Председатель________________ Абрамова И.В.

  • Тема практического занятия – « ».
  • Вид занятия.
  • Время, отведённое на изучение темы.
  • Цели практического занятия (общая и конкретные).
  • Хронологическая карта занятия.
  • Оснащение практического занятия.

Тема занятия:

« Третий этап сестринского процесса – определение целей сестринского вмешательства, планирование ухода ».

Вид занятия: доклиническое практическое

Время отведённое на изучение темы: 90 минут.

Место проведения: кабинет доклинической практики.

Методика проведения занятия: сочетание традиционных

И активных методов обучения.

Цели занятия.

Общие цели:

  • Ознакомить студентов с третьим этапом сестринского процесса.
  • Ознакомить студентов с требованиями к формированию целей.
  • Ознакомить студентов с определением сроков достижения цели.

Конкретные цели:

  • Уметь определить цели сестринских вмешательств;
  • Уметь определить объём сестринских вмешательств;
  • Уметь определить сроки достижения цели;

Хронологическая карта

№ п.п

Этап занятия

Время

(Мин.)

Организационный момент

Проверка присутствующих. Проверка внешнего вида студентов и их готовность к занятиям.

Ознакомление студентов с темой занятия

Определение общих и конкретных целей занятия, знакомство с планом занятия, объяснение терминологии

Введение в тему

Объяснение значения знаний о третьем этапе сестринского процесса: определение целей сестринских вмешательств, их объёмов и сроков достижения.

Контроль исходного уровня знаний

Задание в тестовой форме

Анализ тестового контроля

Оценивается работа каждого студента с выставлением оценки в соответствии с эталонам. Преподаватель вместе со студентами разбирает ошибки и даёт комментарии.

Изучение нового материала

Преподаватель объясняет новый материал, акцентируя внимание студентов на определение целей сестринских вмешательств, их объёмов и сроков достижения.

Самостоятельная работа студентов под руководством преподавателя

Студенты получают задания и выполняют их малыми группами под контролем преподавателя. Группы поочередно выступают с результатами выполнения заданий. Проводится обсуждение хода их решения, коррекция ответов и их оценка.

Заключительный тестовый контроль

Студенты решают задания а тестовой форме. Преподаватель оценивает работу и разбирает ошибки.

Подведение итогов занятия

Преподаватель выставляет среднеарифметическую оценку студентам за работу на занятии:

За работу в малых группах

За решение заключительных тестовых заданий

Задание на дом

Объяснение домашнего задания, инструктаж по внеаудиторной самостоятельной работе студентов.

Итого

Оснащение занятия.

1 . Методическая разработка для преподавателя – 1шт.

2. Методическая разработка для студентов – 10 шт.

3. Задания в тестовой форме для определения исходного уровня знаний – 10 шт.

4. Эталоны ответов для заданий в тестовой форме по определению исходного уровня знаний – 1 шт.

5. Ситуационные задачи для самостоятельной работы студентов малыми группами (по 2 человека) – 10 шт.

6. Эталоны ответов для ситуационных задач – 1 шт.

7. Задания в тестовой форме для текущего контроля – 10 шт.

8. Эталоны ответов для заданий в тестовой форме для текущего тестового контроля знаний студентов и критерии их оценок

– 1 шт.

Блок материалов для обучения:

  • Глоссарий
  • Введение (мотивация темы)
  • Лекционный материал по теме:

«Третий этап сестринского процесса – определение целей сестринского вмешательства, планирование ухода».

  • Источники информации для преподавателя и студента.

Глоссарий

Термин

Определение

Комфорт

Это состояние, при котором человек удовлетворяет все свои потребности.

План ухода

Это письменное руководство по уходу за пациентом, где отражены проблемы, предусмотрены цели и намечены действия по их реализации.

Сестринский уход.

Это создание пациенту комфорта, условий, при которых он может самостоятельно удовлетворять свои потребности.

Цель

Это ожидаемый положительный результат сестринских вмешательств по решению конкретной проблемы пациента. Цели ухода могут быть краткосрочными и долгосрочными. Каждая цель должна состоять из трёх компонентов: действия, критерия и условия.

Холистический подход

Целостный подход – значит лечить не болезнь, а больного.

Введение.

Анализ данных, полученных во время медицинского обследования, даёт основание сестринскому персоналу ставить диагнозы, отражающие индивидуальные реакции пациентов на болезнь или своё состояние. Проблемы выделены, приоритеты в решении определены. Что же дальше?

Следующая ступень сестринского процесса - планирование. Его начинают с определения цели и ожидаемых результатов сестринских действий. В сестринской практике цель - ожидаемый положительный результат сестринских вмешательств по решению конкретной проблемы пациента. Любое действие по уходу за пациентом нацелено на результат, поэтому определение целей необходимо для выбора направления ухода в каждом конкретном случае и для оценки эффективности сестринских действий.

Определение целей ухода и планирование мероприятий по достижению целей - третий этап сестринского процесса. Пациент должен быть активным участником определения целей и планирования ухода.

Цели ухода должны быть реальными, ограниченными по времени, единичными, наблюдаемыми, измеряемыми и в пределах компетенции сестры.

Третий этап сестринского процесса заканчивают составлением плана ухода. План ухода - письменное руководство по уходу за пациентом, где отражены проблемы, предусмотрены цели и намечены действия по их реализации, что уменьшает риск некомпетентного, неверного или неаккуратного ухода. Планы ухода в различных лечебных и образовательных учреждениях имеют отличия в формулировке, но едины, по сути, направлены на достижение определённых целей, в пределах компетенции сестринского персонала как полноправного участника процесса лечения, реабилитации и наблюдения за пациентом.

Пациента надо вовлекать в определение целей и планирование ухода. Только в этом случае его можно признать активным участником сестринского процесса.

Участие пациента в определении целей должно быть так же

независимо, как в определении приоритетов и принятии решений. Задача
сестринского персонала - направлять и защищать пациента, тем самым предотвращая ухудшение его здоровья.

Планирование - мыслительный процесс, заставляющий думать, рассуждать, анализировать. Это прекрасная возможность научиться сопоставлять, объединять, формулировать и фиксировать знания и результаты исследования письменно. Именно поэтому грамотный план ухода - мерило качества работы и профессионализма сестринского персонала.

Опорный конспект.

Третий этап сестринского процесса – определение целей сестринского вмешательства, планирование ухода.

Цели ухода должны отражать не только насущные проблемы, но и меры по реабилитации пациентов и предотвращению развития заболеваний.

Отдельно для каждой приоритетной проблемы формируются цели и план ухода.

Постановка целей необходима по двум причинам:

1. Указание направления для индивидуального сестринского вмешательства.

2. Использование для определения степени эффективности вмешательства.

Каждая цель включает 3 компонента:

1. действие;

2. критерии: диета, время, расстояние;

3. условие: с помощью кого/чего-либо.

Например: в присутствии сестры пациент пройдёт с помощью костылей 10 метров через неделю. Здесь «пациент пройдёт» - действие, в «присутствии сестры с помощью костылей» - условие, «10 метров через неделю» - критерий.

Требования к постановке целей:

1. Цели должны быть реальными, достижимыми.

Нельзя ставить цель: пациент похудеет за 3 дня на 10 кг.

2. Необходимо установить конкретные сроки достижения каждой цели.

Существует 2 вида целей пациентов: краткосрочные и долгосрочные .

Краткосрочные цели должны быть выполнены в короткий период времени, обычно 1-2 недели.

Пример:

  • У пациента будет стул не реже одного раза в 2 дня;
  • У пациента не будет осложнений, связанных с критическим снижением температуры.

Долгосрочные цели могут быть достигнуты к выписке или позже, по возвращению домой, за более длительный промежуток времени.

Эти цели обычно направлены на реабилитацию и предотвращение обострений, приобретение определённых знаний о здоровье. Если сестринский персонал не планирует долгосрочные цели, уход останавливается в день выписки.

Пример:

  • Пациент будет психологически подготовлен к жизни в семье к моменту выписки;
  • Пациент продемонстрирует знания о способах борьбы с запорами к моменту выписки.

3. Цели должны находится в пределах сестринской компетенции.

Неправильно: «У пациента не будет кашля к моменту выписки», т.к. это область компетенции врача.

Правильно: «Пациент продемонстрирует знания о дисциплине кашля к моменту выписки».

4 . Цель должна формулироваться в терминах пациента, а не медсестры.

Неправильно: медсестра обучит пациента технике самовведения инсулина.

Правильно: пациент продемонстрирует умение делать себе инъекции инсулина технически верно через неделю.

5. Пациент должен участвовать в обсуждении каждой цели. Только в этом случае его можно признать активным участником сестринского процесса.

Когда пациент не в состоянии принять участие в определении целей, необходимо по возможности привлекать родственников. К таким состояниям относят: кому, дезориентацию, инсульт с парализацией и потерей речи и т.п. в ряде случаев медицинские сёстры, осуществляющие уход, определяют цели самостоятельно.

Привлекая пациента к определению целей, сестринский персонал готовит себе союзника в борьбе с болезнью. В то же время сестринский персонал, уважая независимость пациента, не навязывает ему свои ценности.

Пример:

  • Одна из целей ухода за пациенткой наркологического диспансера может звучать так: пациентка проявит интерес к жизни к 18.03.

Ожидаемые результаты:

Родственники принимают участие в реабилитации пациентки;

Пациентка читает художественную литературу и строит планы на будущее;

Пациентка чисто и аккуратно выглядит, использует косметику.

6. Цель должна быть единичная, то есть, выводится для каждой ответной реакции на заболевание.

Пример:

Неправильно: «У пациента улучшится аппетит и стабилизируются показатели веса».

Лучше:

Пациент будет съедать весь суточный рацион через 5 дней;

Вес пациента будет стабилизирован к выписке.

7. Цель должна быть наблюдаемая, то есть, результат должен быть очевиден.

Пример:

Медсестра отметит отсутствие пролежней 12.03. пациент сообщит о снижении боли к вечеру. АД: 120/80, PS: 68/мин, ЧДД: 12/мин. Пациент демонстрирует умение самостоятельно вводить себе инсулин 13.03.

8. Цель должна быть измеримая.

Не рекомендовано, определяя результат, употреблять слова: нормальный, удовлетворительный, приемлемый, хороший и пр., так как для разных людей они имеют разное значение. Использование измерительных стандартов (размер, качество, количество, цвет, запах, вес и т. п.) позволяет сестринскому персоналу объективно определить изменения в состоянии пациента и ожидаемый результат.

Пример:

Пациент отметит тенденцию к снижению веса: вес меньше 92 кг;

Пациент отметит увеличение суточного диуреза до 1500 мл;

Пациент и медсестра отметят снижение температуры до 37-37,5 0 С.

После формулировки целей медсестра составляет собственно план ухода за пациентом, то есть письменное руководство по уходу, представляющее собой подробное перечисление специальных действий медсестры, необходимых для достижения целей ухода по приоритетной проблеме пациента.

Медсестра над чистым листом бумаги тщательно обдумывает ситуацию, стараясь подробно, по пунктам, ответить на вопросы - что она может сделать для пациента по данной проблеме? Как облегчить его положение?

ВНИМАНИЕ! План обязательно должен быть конкретным, недопустимы общие фразы и расплывчатые рассуждения. Другими словами, если Вы планируете провести беседу с пациентом, Вы должны записать тезисы этой беседы.

ПЛАНИРОВАНИЕ УХОДА

Формирование целей Действия для

ухода достижение целей

Компоненты целей Виды целей Требования к

формулированию

целей

Действие Краткосрочные Долгосрочные

Реальные;

сосредоточены

Критерий Острая на пациенте;

Профилактика единичные;

обострений наблюдаемые;

реабилитация - совместные;

Условие адаптация измеряемые;

Стадия заболевания

Ожидаемые результаты.

Результат - изменение физиологического, психологического, социального и духовного состояний пациента. Ожидаемые результаты бывают промежуточными и конечными. Промежуточные результаты - ступеньки, ведущие к достижению цели или конечному результату.

Функции ожидаемых результатов.

  • дают направление сестринской деятельности;
  • критерии достижения цели;
  • помогают рассчитать время достижения цели;
  • дают возможность предусмотреть дополнительные ресурсы;
  • критерии оценки сестринских действий.

Ожидаемые результаты вытекают из краткосрочных и долгосрочных целей, сосредоточенных на пациенте и основанных на проблемах, установленных при проведении второго этапа сестринского процесса. Ожидаемые результаты для каждой цели и каждой проблемы должны быть спланированы последовательно, с учётом времени и возможностей пациента. Последовательное перечисление ожидаемых результатов даёт сестринскому персоналу руководство по планированию своих действий.

Ожидаемые результаты записывают в сестринской документации.

Примеры определения целей и ожидаемых результатов

Проблема

Цель ухода

Ожидаемые результаты

Дефицит знаний о подготовке к колоноскопии

Пациент и медсестра отметят, что исследование прошло результативно

Пациент получит необходимые знания от сестры;

Пациент продемонстрирует знания за три дня до исследования;

Пациент уверен в результативности исследования

Риск потери зубов в связи с их подвижностью на фоне парадонтита

Краткосрочная: пациент и медсестра отметят снижение риска потери зубов к концу лечения. Долгосрочная: сохранение зубного ряда как единой динамической системы.

Взаимопонимание между медсестрой и пациентом;

Доверие врачу и желание следовать его рекомендациям;

Регулярное посещение стоматологического кабинета;

Уменьшение подвижности зубов

Существуют три типа сестринских действий, которые можно включить в сестринский план ухода и наблюдения за пациентом: независимые, зависимые и взаимозависимые.

При независимом вмешательстве сестринский персонал действует по собственной инициативе, руководствуясь собственными соображениями и рамками своей компетенции. Это оказание помощи пациенту в самообслуживании, обучение его различным приемам лечения и ухода за собой, организация досуга, советы относительно здоровья, наблюдение за реакциями пациента на болезнь, лечение.

При зависимом вмешательстве действия сестринского персонала осуществляются по назначению врача, часто в его присутствии. Однако и в этом случае не следует автоматически выполнять указания врача. Уточнение назначений входит в компетенцию сестриного персонала. Сестринский персонал, выполнивший неверное или ненужное предписание, профессионально некомпетентен, и в той же степени, что и врач, ответственен за последствия.

При взаимозависимом вмешательстве сестринский персонал сотрудничает с другими специалистами на равных (например, реанимационные мероприятия).

Методы сестринских вмешательств, т.е., какого рода сестринская помощь может планироваться медицинской сестрой.

Сестринская помощь планируется на основе нарушенных потребностей пациента, с целью их удовлетворения.

Методы сестринских вмешательств могут являться и способами удовлетворения нарушенных потребностей.

  • Оказание доврачебной помощи.
  • Выполнение врачебных назначений.
  • Создание комфортных условий для жизнедеятельности с целью удовлетворения основных потребностей пациента.
  • Оказание психологической поддержки и помощи.
  • Выполнение технических манипуляций.
  • Мероприятия по профилактике осложнений и укреплению здоровья.
  • Организация обучения и консультирования пациента и членов его семьи.

При составлении плана ухода медсестра может руководствоваться соответствующим СТАНДАРТОМ сестринского вмешательства, если под стандартом понимать перечень научно обоснованных мероприятий, обеспечивающих качественный уход за пациентом по определённой проблеме.

Например, ознакомьтесь с примерным СТАНДАРТОМ сестринских вмешательств при проблеме «стул со склонностью к запорам».

Сестринская проблема: стул со склонностью к запорам.

Цели:

  • Краткосрочная – у пациента будет стул не реже одного раза в два дня (время индивидуально).
  • Долгосрочная – пациент продемонстрирует знания о способах борьбы с запорами к моменту выписки.

Характер сестринского вмешательства:

1. Обеспечить кисло-молочную растительную диету (творог, овощи, чёрный хлеб, фрукты, зелень) – диета № 3.

2. Обеспечить достаточный приём жидкости (кисол-молочные продукты, соки, сульфатные минеральные воды) до 2-х литров в день.

3. Попытаться выработать у пациента условный рефлекс на дефекацию в определённое время суток (утром через 15-20 минут после приёма стакана холодной воды натощак).

4. Обеспечить достаточную двигательную активность пациенту.

5. Обеспечить приём слабительных и постановку опорожнительных клизм по назначению врача.

6. Регистрировать ежедневно кратность стула в медицинской документации.

7. Обучить пациента особенностям питания при запорах. Рекомендовать расширение режима двигательной активности.

Стандарт создаётся в помощь медсестре, это справочник, но следует помнить, что в стандарте невозможно предусмотреть всё разнообразие клинических ситуаций, поэтому нельзя применять необдуманно, слепо. ещё Пётр I предостерегал: «Не держись за устав, как слепой за плетень».

Пример, включение в рационе большого количества овощей и фруктов, чёрного хлеба не может быть рекомендовано страдающему запорами пациенту с воспалительным заболеванием кишечника, много жидкости, постановки очистительных клизм объёмом в 1,5 – 2л – пациенту с запорами на фоне отеков, а расширение двигательной активности - пациенту с запорами и травмой позвоночника.

Формальные стандарты облегчают работу медсестры, но никогда не заменят ее самостоятельного суждения. Стандарты обобщают сестринские знания и рассчитаны лишь на работу в ТИПИЧНОЙ ситуации - со среднестатистическим, а не с конкретным пациентом. От медсестры при составлении индивидуального плана ухода требуется умение гибко применять стандарт в реальной практической ситуации. Медсестра имеет право исключить любой пункт стандарта из своего плана ухода, либо, наоборот, дополнить план действиями, непредусмотренными стандартом, но она должна уметь аргументировать свою точку зрения.

Акцент сестринского вмешательства - это всегда дефицит способности пациента поддерживать свое состояние, то есть функционировать независимо.

Составление и ведение индивидуальных планов ухода имеет ряд преимуществ перед привычной практикой работы сестринского персонала:

  • план ухода - письменное руководство по уходу за пациентом, где отражены проблемы, предусмотрены цели и намечены действия по их реализации, что уменьшает риск некомпетентного, неверного или неаккуратного ухода;
  • план координирует уход, экономит время и способствует
    эффективному использованию оборудования и материальных ресурсов, так как любая медсестра может быстро определить, что нужно пациенту;
  • план определяет продолжительность ухода за пациентом и степень участия в нём каждого медработника;
  • план ухода представляет собой письменный обмен информацией в форме отчётов;
  • письменный план ухода, где предусмотрены долгосрочные цели, способствует обеспечению непрерывности ухода и его продолжительности после госпитализации или перевода пациента в другое Л ПУ;
  • план позволяет быстро подсчитать материальные затраты на уход.

К разработке планов ухода сестринский персонал подключает пациента и его родственников. Семья часто - основной рычаг в достижении целей ухода.

Результат планирования - «живой» план ухода, где отражена динамика самочувствия пациента, цели и объём оказанной сестринской помощи, предусматривающий все виды сестринских вмешательств и реакцию пациента на уход. Планы ухода в различных лечебных и образовательных учреждениях имеют отличия в формулировке, но едины по сути, направлены на достижение определённых целей в пределах компетенции сестринского персонала как полноправного участника процесса лечения, реабилитации и наблюдения за пациентом.

Источники информации.

1. И.Х. Аббясов, С.И. Двойников, Л.А. Карасёва «Основы сестринского дела». Учебник. – М.: Издательский центр «Академия», 2007.

2. С.А. Мухина, И.И. Тарновская. «Теоретические основы сестринского дела» Учебник. М.: Родник, 1998 г.

3. И.В. Островская, Н.В. Широкова. «Основы сестринского дела» Учебник. – М.: ГЭОТАР-Медиа, 2008 г.

4. Т.П. Обуховиц, Т.А. Склярова, О.В. Чернова. «Основы сестринского дела». Учебное пособие. Ростов на Дону. «Феникс» 2007.

5. Г.М. перфильева, Н.Н. Камынина, И.В. островская, А.В. Пьяных. «Теория сестринского дела» Учебник – М.: «ГЭОТАР – Медиа» 2009.

6. А.И. Шпирин. Учебно-методическое пособие по основам сестринского дела. М.ВУМНУ. 2000.

  • Задания в тестовой форме для контроля исходного уровня знаний студентов, эталоны ответов к ним, критерии оценки;
  • Задания для самостоятельной работы студентов малыми группами по 2 человека, эталоны ответов к ним, критерии оценки;
  • Задания в тестовой форме для текущего контроля знаний студентов, эталоны ответов ним, критерии оценки;
  • Вопросы для самоподготовки студентов.

Задания в тестовой форме

для проведения контроля исходного уровня знаний студентов.

Заполнить пробелы:

1. В некоторых случаях медсестра определяет цель ________________, без

Участия пациента.

2. Долгосрочные цели обычно направлены на _______________ осложнений.

3. Цель должна быть_______________________, т.е. отражать одну проблему пациента.

4. Цель должна быть ____________________ по времени.

5. _________________________ - базовый перечень научно обоснованных мероприятий по уходу, обеспечивающих достижение целей по проблеме независимо от конкретной ситуации.

Ответьте -

верно или неверно:

1. Начало третьего этапа сестринского процесса – составление плана ухода.

2. Цели ухода должны быть сосредоточены на пациенте.

3. Цели ухода должны объединять несколько проблем пациента.

4. Формулировка цели состоит из двух компонентов.

5. Стандартный план ухода представляет собой мероприятия по решению стандартных проблем пациента без учёта конкретной ситуации.

6. Краткосрочные цели должны быть достигнуты в течение первых суток ухода.

7. План ухода – это критерий оценки работы сестринского персонала.

8. Планы ухода в обязательном порядке должны быть согласованы с лечащим врачом.

9. Родственников нельзя привлекать к составлению плана ухода.

10. Цель ухода за пациентом, страдающего запором, должна звучать так: «Поставить очистительную клизму».

Эталоны ответов

к тестовому контролю по исходному уровню знаний

По теме: «Третий этап сестринского процесса»

Заполнить пробелы:

1. Самостоятельно.

2. Профилактику.

3. Единичной.

4. Краткосрочной.

5. Стандартный.

Ответьте верно или неверно:

1. Неверно.

2. Верно.

3. Неверно.

4. Неверно.

5. Верно.

6. Неверно.

7. Неверно.

8. Неверно.

9. Неверно.

10. Неверно.

Критерии оценок:

5 баллов – (100-91%) – 0-1 ошибка

4 балла – (90-81%) – 2-3 ошибки

3 балла – (80-71%) – 4-5 ошибок

2 балла – (70 и менее %) более 5 ошибок

Инструкция

для самостоятельного выполнения обучающих заданий по текущему контролю знаний студентов

по теме: «Третий этап сестринского процесса»

  • Студентам разделиться на малые группы по 2 человека, разделить роли. Один студент получает роль медсестры, другой роль эксперта, затем меняются ролями.
  • Каждая группа студентов получает задания с описанием ситуации, при которой необходимо составить план оказания сестринской помощи пациенту согласно ранее выявленной проблемы.
  • На выполнение задания отводится 10 минут.
  • Работу малых групп корректирует и оценивает преподаватель.

Ситуационные задачи

для самостоятельной работы студентов

по осуществление третьего этапа сестринского процесса.

Задача № 1.

Пациент Сорокин Алексей Петрович 45 лет находится на лечении в

хирургическом пульмонологическом отделении.

Пациента беспокоит периодический кашель, сопровождающийся отхождением

большого количества зловонной мокроты.

Вес 55кг, рост 175см. ЧДД - 22 в минуту, ЧСС -106 в минуту, АД 110\60 мм рт.ст., t° тела

37,2°.

Пациенту назначено принимать 4 раза в день по 40 минут дренажное положение, но А.П. не хочет этого делать, т.к. ему очень неудобно лежать в таком положении, и "мокрота и так хорошо отходит".

Проблема: пациент не хочет принимать положение, улучшающее отхождение мокроты (дренажное положение).

Задание:

Задача № 2.

Пациент Данилов А. В. 48 лет поступает на лечение в пульмонологическое

отделение с обострением бронхо-легочного заболевания.

Первичная оценка состояния пациента:

Пациента беспокоит частый кашель с серозно-гнойной мокротой, которую пациент

собирает носовой поток или сплевывает раковину.

Вес 75кг, рост 175см. ЧДД - 20 в минуту, ЧСС -106 в минуту, АД 110\60 мм рт.ст.,

t °тела - 38,5° утром и 39,4 вечером. Курит в течение многих лет по 10-15 сигарет в день.

Проблема: пациент не пользуется плевательницей.

Задание: сформулировать цель сестринского вмешательства и составить план ухода (письменно).

Задача № 3.

Пчелкина Елена Борисовна 32 года, находится на лечении в терапевтическом отделении.

Первичная оценка состояния пациента:

ЧДД - 20 в минуту, ЧСС - 82 в минуту, АД -130\85, t ° тела - 36,7°. Рост -165см, вес - 70кг.

Пациентку беспокоит периодически одышка, возникающая на различные запахи. Врачом назначен индивидуальный ингалятор, которым пациентка не умеет и боится пользоваться.

Проблема: пациент не умеет пользоваться карманным ингалятором.

Задание: сформулировать цель сестринского вмешательства и составить план ухода (письменно).

Задача № 4.

Дегтярёв Семён Давидович 39 лет, госпитализирован в связи с обострением хронического панкреатита. Заболел несколько часов назад, когда поднялась температура тела до 38,9°С.

Первичная оценка состояния пациента:

ЧДД - 22 в минуту, ЧСС - 97 в минуту, АД - 125\85 мм рт.ст., t ° тела - 39,0°С. Рост - 154 см, вес - 58 кг.

В настоящий момент температура сохраняется. Пациент жалуется на чувство жара, повышенное потоотделение. Укрыт двумя байковыми одеялами, одет в байковую пижаму. Пациент не понимает необходимости укрыться более лёгким одеялом, сменить одежду. Врач рекомендовал измерять температуру тела у пациента каждые 2 часа.

Проблема: пациент не понимает необходимости укрыться более лёгким одеялом и сменить одежду.

Задание: сформулировать цель сестринского вмешательства и составить план ухода (письменно).

Задача № 5.

Волгапкин Сергей Александрович, 47 лет, находится на лечении в терапевтическом отделении по поводу острого бронхита.

Первичная оценка состояния пациента:

ЧДД -в 23 минуту, ЧСС - 92 в минуту, АД - 140\85мм рт.ст., t ° тела - 37,5°С. Рост - 193 см, вес - 82 кг.

Пациента беспокоит головная боль, ломота в костях, озноб. Кожные покровы бледные, холодные, цианоз губ. Пациент жалуется на жажду, но самостоятельно приготовить питьё не может.

Проблема: пациент не может оказать себе помощь во время озноба.

Задание: сформулировать цель сестринского вмешательства и составить план ухода (письменно).

Задача № 6.

Алексеева М.П., 60 лет, пенсионерка, находится на лечении в терапевтическом отделении по поводу повышения артериального давления.

Первичная оценка состояния пациента:

ЧДД - 20 в минуту, ЧСС - 87 в минуту, АД - 180/100 мм.рт.ст., t ° тела - 36,7°С Рост - 163 см., вес - 105 кг.

Пациентка всегда любила соленые и жирные блюда, диету никогда нее соблюдала, несмотря на то, что её всегда беспокоил избыточный вес. Жидкости пьет много до 2,5 литров в сутки. Много времени просиживает у телевизора - смотрит сериалы. На улицу выходит только для того, чтобы дойти до ближайшего магазина. Водный баланс не определялся. Алкоголь не употребляет. Зубы имеются полностью.

Проблема: вес пациентки значительно превышает норму.

Задание: сформулировать цель сестринского вмешательства и составить план ухода (письменно).

Задача № 7.

Пациент 43-х лет, находящийся на стационарном лечении по поводу хронического бронхита, предъявляет жалобы на нарушение сна - с первых дней пребывания в больнице не может заснуть до 2-3 часов утра, оставшееся время дремлет с перерывами. Утром чувствует себя разбитым, раздраженным, болит голова. Раньше проблем со сном не было, связывает их появление с переменой обстановки (госпитализация). Просит у медсестры снотворное.

Проблема: пациент не может заснуть из-за того, что попал в незнакомую обстановку.

Задание: сформулировать цель сестринского вмешательства и составить план ухода (письменно).

Задача № 8.

Пациент 45 лет после холецистэтомии находится на постельном режиме. При использовании мочеприёмника стесняется соседей по палате.

Проблема: пациент стесняется соседей по палате при использовании мочеприёмника.

Задание: сформулировать цель сестринского вмешательства и составить план ухода (письменно).

Задача № 9.

Пациент находится на лечении по поводу обструкции верхних дыхательных путей. Произведено оперативное вмешательство – трахеотомия. Пациент понимает о чём ему говорят, но ответить не может. Из-за этого переживает, раздражается.

Проблема: затруднено вербальное общение в связи с трахеостомой.

Задание: сформулировать цель сестринского вмешательства и составить план ухода (письменно).

Задача № 10.

Сидоркин Константин Петрович 62 лет находится на лечении в терапевтическом отделении.

Первичная оценка состояния пациента:

ЧСС - 88 в минуту, аритмичный, ЧДД - 22 в минуту, АД - 140\90мм рт.ст., Рост - 170см, вес - 85кг. t ° тела - 36,5°.

Предпочитает лежать на кровати с высоко поднятым изголовьем. Двигается только по необходимости. Риск развития пролежней по шкале Ватерлоу 8 баллов. Встает с кровати редко, с помощью сестры или родственников. Разрешено двигаться в пределах палаты с помощью ходунков, но пациент не умеет ими пользоваться.

Проблемы:

- трудности при ходьбе, при расширении режима активности;

- пациент не умеет пользоваться ходунками.

Задание: сформулировать цель сестринского вмешательства и составить план ухода (письменно).

Критерии оценки знаний – умений студентов при решении ситуационных задач.

Оценка знаний – умений студентов проводится по 5-ти бальной шкале.

«5» (отлично)

Чётко сформулированы цели сестринского вмешательства, правильно составлен план ухода.

«4» (хорошо)

Чётко сформулированы цели сестринского вмешательства, план ухода не полностью обеспечивает достижение целей. Ответ неуверенный.

«3» (удовлетворительно)

Не чётко сформулированы цели сестринских вмешательств. План ухода не обеспечивает достижение целей, скорректирован с помощью наводящих вопросов преподавателя.

«2» (неудовлетворительно)

Цели сестринских вмешательств сформулированы не правильно. План ухода не отражает достижение поставленных целей.

Эталон ответа к задаче № 1.

План ухода.

Проблема

Цели с/в краткосрочные

Сестринское вмешательство

Текущая оценка

Ожидаемый результат

Пациент не хочет принимать положение,

улучшаю-

щее отхожде-

ние мокроты.

Пациент принимает положение, улучшающее отхождение мокроты (дренажное положение)

1. Объяснять необходимость принимать дренажное положение.

2. Показывать, какое дренажное положение должен принимать пациент.

3. Помогать пациенту принимать дренажное положение.

4. Следить за тем, чтобы пациент самостоятельно принимал дренажное положение.

5. Следить за наполнением плевательницы и регулярно менять её.

4-5 раз в день по мере необходимости принимать данное положение.

Пациент принимает самостоятельно положение, улучшающее отхождение мокроты (дренажное положение).

Эталон ответа к задаче № 2

План ухода.

Проблема

Цели с/в краткосрочные

Сестринское вмешательство

Текущая оценка

Ожидаемый результат

Пациент не пользуется плевательницей

Пациент пользуется плевательни-

цей.

1. Объяснять пациенту, почему необходимо пользоваться плевательницей.

2. Обучить пациента пользоваться плевательницей.

3. Обеспечить пациента чистыми плевательницами.

4. Следить за наполнением плевательницы.

3-4 раза в день во время смены плевательницы.

Пациент пользуется плевательницей через день.

Эталон ответа к задаче № 3

План ухода.

Проблема

Цели с/в краткосрочные

Сестринское вмешательство

Текущая оценка

Ожидаемый результат

Пациентка не умеет и боится пользоваться карманным ингалятором.

Пациентка пользоваться карманным ингалятором.

1. Обучить пациента пользоваться карманным ингалятором.

2. Дать памятку.

Через 30 минут.

Через 24 часа пациент пользуется карманным ингалятором.

Эталон ответа к задаче № 4

План ухода.

Проблема

Цели с/в краткосрочные

Сестринское вмешательство

Текущая оценка

Ожидаемый результат

Пациент не понимает необходимости укрыться более лёгким одеялом и сменить одежду.

1. Пациент укроется более лёгким одеялом с помощью сестры/родственника.

2. Пациент сменит одежду.

3. Температура тела будет снижаться.

1. Объяснить необходимость укрыться более лёгким одеялом.

3. Измерять температуру тела через каждые 2 часа.

4. Регистрировать результаты измерения температуры тела в температурном листе.

5. Осуществлять процедуры, увеличивающие теплоотдачу (пузырь со льдом, холодный компресс, вентилятор, обтереть кожу губкой, обёртывание во влажную простынь.

6. Вводить лекарственные средства по назначению врача.

7. Консультация врача при любом ухудшении состояния и самочувствия пациента.

По мере необходимости.

По мере необходимости.

Каждые 2 часа.

Каждые 2 часа.

Не менее 5 раз в день.

По назначению врача.

По необходимости.

Пациент укроется лёгким одеялом.

Пациент сменит одежду.

Температура снизится на 1 0 С.

Эталон ответа к задаче № 5

План ухода.

Проблема

Цели с/в краткосрочные

Сестринское вмешательство

Текущая оценка

Ожидаемый результат

Не может оказать себе помощь во время озноба.

Помощь будет оказана медицинской сестрой/родственниками.

1. Измерение температуры тела.

3. Обеспечить горячим питьём (чай с лимоном, малиновым вареньем).

Однократно.

По мере необходимости.

Пациент чувствует себя комфортно.

Эталон ответа к задаче № 6

План ухода.

Проблема

Цели с/в краткосрочные

Сестринское вмешательство

Текущая оценка

Ожидаемый результат

Вес пациентки значительно превышает норму.

1. Масса тела пациентки уменьшится на 2 кг за неделю.

2. Пациентка исключит из рациона солёные, острые, жирные блюда.

1. Обсудить с пациенткой последствия повышенной массы тела.

2. Попросить родственников оказывать пациентке моральную поддержку.

3. Рассказать родственникам о характере передач.

4. Контроль передачи.

5. Объяснить пациентке принципы диеты для снижения веса.

6. Обратиться к диетологу.

7. Взвешивать один раз в неделю – утром.

Однократно.

Однократно.

Однократно.

Ежедневно.

Ежедневно.

Однократно.

1 раз в неделю.

Через неделю масса тела снизилась на 2 кг.

Пациентка исключила из рациона солёные, острые, жирные блюда.

Эталон ответа к задаче № 7

План ухода.

Проблема

Цели с/в краткосрочные

Сестринское вмешательство

Текущая оценка

Ожидаемый результат

Пациент не может заснуть (часто просыпается) из-за того, что попал в незнакомую обстановку

1. Тревожность пациента уменьшится.

2. Пациент будет говорить, что высыпается к концу недели.

1. Ознакомить пациента с месторасположением туалета, сестринского поста.

2. Обеспечить средством связи с сестрой.

3. По просьбе пациента сообщить родственникам.

4. Побуждать пациента выражать своё беспокойство.

5. Быть готовой выслушивать пациента и дать ему совет, а также оказать ему психологическую поддержку.

6. Стараться обеспечить пациенту спокойную и комфортную для него окружающую обстановку, принимая во внимание его индивидуальные особенности и просьбы.

7. Дать лекарственные препараты (снотворные, обезболивающие) по назначению врача.

8. Убедиться, что пациент осознаёт важность сна для скорейшего выздоровления.

Однократно.

Однократно.

Однократно.

Ежедневно.

Ежедневно.

Ежедневно.

По мере необходимости.

Однократно.

Тревожность пациента уменьшилась.

Пациент говорит, что высыпается.

Эталон ответа к задаче № 8

План ухода.

Проблема

Цели с/в краткосрочные

Сестринское вмешательство

Текущая оценка

Ожидаемый результат

Пациент стесняется соседей по палате при использовании судна, мочеприёмника.

1. Обеспечить возможность уединения (ширма, попросить выйти соседей по палате, накрыть одеялом).

2. Своевременно приходить на вызов.

3. Своевременно опорожнять и дезинфицировать судно и мочеприёмник.

4. Обучить родственников помогать пациенту использовать судно и мочеприёмник.

5. Создать условия для самостоятельного использования судна и мочеприёмника.

6. Психологическая поддержка.

5-6 раз в день при каждой подаче судна.

Пациент не стесняется соседей по палате.

Эталон ответа к задаче № 9

План ухода.

Проблема

Цели с/в краткосрочные

Сестринское вмешательство

Текущая оценка

Ожидаемый результат

Затруднено вербальное общение в связи с трахеостомой.

1. Прикрепить рядом с кроватью плакат с картинками (пища, питьё, судно, мочеприёмник и т.д.).

2. Обеспечить пациента ручкой, бумагой.

3. Обсудить с пациентом способы невербального общения, обозначающие «да», «нет» и др.

Однократно.

Однократно.

Однократно.

Персонал, родственники и пациент понимают друг друга.

Эталон ответа к задаче № 10

План ухода.

Проблема

Цели с/в краткосрочные

Сестринское вмешательство

Текущая оценка

Ожидаемый результат

1. Трудности при ходьбе при расширении режима активности.

2. Пациент не умеет пользоваться ходунками.

Ходит в пределах палаты.

Пациент использует ходунки.

1. Обучить родственников технике правильного безопасного перемещения пациента.

2. Поддерживать при ходьбе.

3. Обучить пациента пользоваться дополнительными приспособлениями.

4. Обеспечить дополнительными приспособлениями.

1. Обучить пациента пользоваться специальными приспособлениями.

2. Оказывать необходимую помощь при передвижении.

4 раза в день.

По индивидуальной программе.

Ходит в пределах палаты.

Пациент использует ходунки.

Задания в тестовой форме

Выбрать один ответ:

1. Краткосрочные цели ухода должны быть выполнены за:

а) 1-2 нед;

б) 3 нед;

в) 1 мес.

2. Правильно поставленная цель сестринских действий:

а) у пациента не будет недержания мочи;

б) пациент отметит улучшение самочувствия;

в) пациент продемонстрирует умение использовать съёмный
мочеприемник через день;

г) сестра обучит пациента правилам применения мочеприемника.

3. Цели ухода ставят:

а) только медицинская сестра;

б) медицинская сестра и врач;
в) врач и пациент;

г) пациент и медицинская сестра.

4. Оценочные критерии эмоционального статуса пациента:

а) активность, координация;

б) уровень образования, способность решать проблемы;

в) материальное обеспечение, отношение с окружающими;

г) представление о себе, поведение;

д) верования, ценности.

5. Третий этап сестринского процесса заканчивают:

а) определением приоритетов ухода;

б) постановкой целей и определением ожидаемых результатов ухода;

в) составлением плана ухода;

г) выполнением плана.

6. Формулировка цели ухода должна быть сосредоточена на:

а) ожиданиях пациента;

б) действиях сестры;

в) действиях врача.

7. Стандартный план ухода представляет собой:

а) базовый уровень обслуживания без учёта конкретной ситуации;

б) подробное описание проблем пациента с учётом конкретной ситуации.

8. Цель сестринского процесса:
а) диагностика заболеваний; б) управление лечением;

в) установление и поддержание независимости пациента.

9. В план ухода по решению проблемы пациента «Дефицит самоухода, связанный с необходимостью соблюдать постельный режим» медсестра введёт:

а) обеспечение щадящего режима перестилания и перекладывания

пациента;

б) обучение пациента самоуходу в условиях ограничения подвижности;

в) обеспечение физического и психологического покоя; беседы о необходимости лечения.

10. Ожидаемый результат при решении проблемы пациента «Риск истощения, по причине анорексии, вызванной депрессией»:

а) увеличение массы тела;

б) пациентка съедает половину суточного рациона;

в) пациентку кормят через желудочный зонд.

11. Требования к постановке целей - все, кроме: а) должны иметь гибкие условия выполнения; б) цели должны быть реальными;

в) должны формулироваться в научных медицинских терминах; г) должны иметь конкретные критерии и условия выполнения; д) цели должны находится в пределах сестринской компетенции.

12. По приоритетной проблеме "сильная головная боль" (у пациента с поднимающейся температурой) цель может быть сформулирована следующим образом:

а) у пациента снизится температура до 37°С через 5 часов; б) у пациента не будет головной боли через некоторое время; в) поддерживать температуру на уровне 39°С до прихода врача; г) у пациента снизится температура через 5 часов; д) поддерживать боль на уровне терпимости до консультации врача.

13. Независимый тип сестринского вмешательства: а) очистительная клизма; б) смена повязок; в) промывание желудка; г) экстренные меры по спасению жизни пациента; д) профилактика пролежней.

14. Зависимый тип сестринского вмешательства:

а) мероприятия по личной гигиене пациента; б) обучение пациента;

в) дыхательная гимнастика;

г) оказание психологической помощи;

д) инъекция обезболивающего.

15. Пациент испытывает тупую боль внизу живота справа. Как вы сформулируете цель ухода по этой проблеме до консультации врача?

а) боль будет облегчена после приема но-шпы через полчаса;

б) пациент не будет испытывать сильного волнения по поводу

неизвестной причины болей;

в) боль должна пройти после постановке клизмы за счет очищения

кишечника;

г) пациент будет чувствовать себя хорошо после применения но-шпы

через полчаса.

16. При приоритетной проблеме сильная боль в горле (у пациента,

страдающего ангиной) цель сестринского ухода будет звучать

следующим образом:

а) через сутки у пациента не будет болеть горло;

б) надо дать лекарство от ангины и создать максимальный комфорт;

в) у пациента облегчится чувство боли в горле, благодаря полосканию

через каждые 2 часа;

г) пациенту дать снотворное и он не будет чувствовать боли в горле.

17. При приоритетной проблеме "неадекватное поведение в состоянии

алкогольного опьянения" (у пациента, страдающего запойным

пьянством) краткосрочная (1) и долгосрочная (2) цели будут звучать так:

а) члены семьи пациента будут владеть следующими навыками:

грамотного общения с пьяным человеком, вытрезвления в домашних

условиях;

б) пациент будет избавлен от алкогольной зависимости методом

кодирования через 1 неделю;

в) пациент будет вести трезвый образ жизни;

г) пациент при поддержке семьи осознает преимущества трезвого образа

жизни и даст согласие на лечение.

18. План ухода мед. сестра согласовывать с врачом а) должна; б) не должна.

Заполнить пробелы:

19. ________________________________ - это диагностика и лечение реакций организма на существующие или потенциальные проблемы со здоровьем.

20. ________________________________ - это выявление конкретного заболевания или патологического процесса.

Эталоны ответов к заданию в тестовой форме

по теме «Третий этап сестринского процесса»

для текущего контроля знаний студентов.

1. – а

2. – в

3. – г

4. – а

5. – в

6. – а

7. – а

8. – в

9. – б

10. – а

11. – в

12. – д

13. – д

14. – д

15. – б

16. – а

17. – 1а, 2г

18. – б

19. – сестринский диагноз

20. – врачебный диагноз

Критерии оценок:

5 баллов – (100-91%) – 1-2 ошибки

4 балла – (90-81%) – 3-4 ошибки

3 балла – (80-71%) – 5-6 ошибок

2 балла – (70 и менее %) 7 и более ошибок.

Вопросы для самоподготовки студентов